You are on page 1of 54

• CHAPTER 09 .

Ray Optics and

R
Optical Instruments

SI
Chapter Analysis w.r.t: Lost 3 Yeor's Boord Exoms
The analysis given here gives you an analytical picture of this chapter and will help you to identify the
concepts of the chapter that are to befocussed morefrom exam point of view.
IT
Number of Questions asked in last 3 years
2015 2016 2017
Delhi All India Delhi All India Delhi All India
Very Short Answer (l mark) 1Q lQ 1Q
Short Type I Answer (2 marks) lQ 1Q
H
Short Type II Answer 13 marks) lQ lQ lQ lQ
Long Answer 15 marks) lQ lQ
Value Based Questions (4 marks)
• In 2015, in Delhi set, one question of 1 marks based on Scattering of Light , one numerical
question of 2 marks based on Magnifying Power of Telescope and one question of 5 marks
O

based on Angle of Deviation, Dispersion of Prism and Reflecting Index of Glass were asked.
In All India set, one question of 1 mark based on Refractive Index, one question of 2 marks I
based on Mirror Formula were asked.
• In 2016, in Delhi set, only one numerical question of 3 marks based on Prism Formula was
asked. In All India set, one question of 3 marks based on Prism Formula and one question of 5
M

marks based on Spherical Refracting Surface and Its Assumptions, Lens Maker's Formula
and numerical based on it were asked.
• In 2017, in Delhi set only one numerical question of 3 marks based on Prism Formula was
asked. In All India set, one question of 1 mark based on Angular Deviation, one numerical
question of 3 marks based on Critical Angle were asked.
On the basis of above analysis, it can be said that from exam point of view Angle of Deviation,
Dispersion of Prism, Reflecting Index of Glass, Prism Formula and Lens Maker's Formula are most
important concepts of the chapter.
[TOPIC 1] Reflection, Refroction
and Dispersion of Light
Ray Optics or Geometrical Types of Mirror
Optics Plane Mirror

R
In this branch of optics, the light is In a plane mirror, image formed is always virtual, erect
considered as a ray which travels in a equal in size as that of the object and at the same distance
straight line. behind the mirror as the object is in front of the mirror.
It states that for each and every object, there Image in a plane mirror is always laterally inverted.
is an image. It deals with the phenomena of

SI
reflection and refraction of light by ordinary Spherical Mirror
geometrical methods. A type of mirror whose reflecting surfaces is part of a
hollow sphere. Spherical mirrors are of two types.

1.1Reflection (i) Concave spherical mirror


(ii) Convex spherical mirror
Reflection is the phenomenon of changing
Polished
the path of light after incidenting on a
boundary separating two media without any
IT convex(~p: Surface
change in the medium. mirror Concave
mirror

Reflection of Light
The returning back of light in the same
, E/, /

< , ---- Retlectinq surface


medium from which it has come after
H
striking a surface is called reflection of light.
1.2 Sign Convention
Laws of Reflection
All measurements should be taken from pole of mirror.
Two laws of reflection are given as below:
• All measurements along the direction of incident ray will
O

;N(Normal)
be positive and opposite to incident ray are negative.
I
I
I • All the measurements for the distances above the
I
I principal axis are taken as positive and below the
I
I principal axis are taken as negative.
i
M

I
r • For a real object, u is negative whereas v is negative for
rrm7T1Tl7T1Tlmn:rmr.~7T1TI7T1TImn:rmr.rmn Mirror real image and positive for virtual image.
o Mirror
Heights
(i) The angle of incidence i is equal to the
upward
angle of reflection r. positive P(Pole)
i.e. Li = Zr.
Heights X-axis
(ii) The incident ray, reflected ray and downwards
normal to the reflecting surface at the negative
Distances along
point of incidence all lie in the same incident light
plane. positive
CHAPTER 9 : Roy Optics And Optical Instruments 253

Normal
1.3 Mirror Formula
Mirror formula is a relation between focal length (Rarer)
Air
of the mirror, distances of objects and image from
the mirror. Glass
(Denser)
1 1 1
-+-=-
v u f

R
(c)
where, f = focal length, u = distance of the object
Two laws of refraction are given as below:
from mirror, v = distance of the image from mirror.
(i) The incident ray, refracted ray and the
Focal length of mirror normal to the refracting surface at the point
(f) = Radius of curvature (R) => f =~ of incidence lie in the same plane.

SI
2 2 (ii) The ratio of the sine of the angle of
incidence to the sine of the angle of
refraction is constant for the two given
1.4 Linear Magnification media. This constant is denoted by ~ and is
called the relative refractive index.
The ratio of the size of the image formed by a
spherical mirror I to the size of the object 0 is 1~2 = s~ i (Snell's law)
called the linear magnification produced by the smr
spherical mirror. where, ~ is refractive index of the second
IT
I v f f-v medium with respect to first medium.
m=-=--=--=--
o u f-u f
Refractive Index in Terms of
where, I = height of image and 0 = height of
object
Wavelength
The refractive index (u) of a material is the ratio of
H
Magnification (m) the speed of light (c) in vacuum to the speed of
It is negative corresponding to real image and light in the medium (v).
positive for virtual image. Mathematically, if refractive index is given by the
. ~ = --0..
re Iation Speed__
of---'''--
light in the vacuum_ c
O

Speed of light in the medium v


1.5 Refraction
It is also referred to as absolute refractive index
The phenomenon of changing in the path of light of the substance.
as it goes from one transparent medium to
another is called refraction.
Principle of Reversibility of Light
M

N
Laws of Refraction Principle of reversibility I
I
of light states that when I
I
I
Incident ray Normal Incident ray final path of a ray of A I
I

light after suffering any •


Z
I
I
I

number of reflections
(Rarer)
Air and refractions is
reversed, the ray retraces
Air
(Rarer) its path, exactly.
L..---N~,;---"-",,aRe
=> 1~2 X 2~1 =1 mirror
Refracted ray 1 Principle of reversibility of light
(b) 2~1=-
1~2
254 [21 Chapterwise CSSE Solved Papers PHYSICS

Critical Angle (ic)


It is the angle of incidence in denser medium for
which angle of refraction in rarer medium is 90° is
called the critical angle of the denser medium.

111
Critical ic Denser medium 1J.2_h=1J.2-1J.1
angle

R
V u R
Rarermedium
Il2 where IJ.I' IJ.2 are refractive index of rarer
and denser media and u, v and R are to be

1
w~e~ i = ic,r = 90° 1 taken with their proper signs.

SI
i.e. IJ.=- sinr, 1 (ii) When refractiontakes place from denser to
sin t, [ -'--0 =21J.1=- rarer medium, then
sm90 lIJ.2
IJ.I _1J.2 = IJ.I -1J.2
where, IJ.= refractive index of denser medium v u R
w.r.t. rarer medium.

1.5 Total Internal


1.7 Lens
Lens is a transparent medium bounded by two
Reflection
IT
(TIR) surfaces of which one or both surfaces are
spherical.
When a ray of light travelling from denser
medium to rarer medium is incident at the
interface of two medium at an angle greater than
Convex or Converging Lens
the critical angle for the two media, the ray is A lens which is thicker at the centre and thinner
H
totally reflected back to denser medium. This at its end is called convex lens. Convex lenses are
phenomenon is called Total Internal Reflection. It of three types which are given as below:
occurs only when angle of incidence in denser

))
medium is greater (not equal) than critical angle,
i.e. i» i..
O

Rarer ~ ~
medium
(Air)
Double convex Plano-convex Concavo-convex
lens lens lens
M

Denser
medium
Concave or Diverging Lens
(Water) Partially
C reflected rays A lens which is thinner at the centre and thicker
A
Total Internal Reflection at its ends is called a concave lens. Concave lenses
are of three types which are given as below:

1.6 Refraction at a
Spherical Surface
(i) Refraction formula for refraction by convex or
concave spherical refracting surface is given Double Plano-concave Convexo-concave
by concave lens lens lens
CHAPTER 9 : Roy Optics And Optical Instruments 255

(ii) If 11' =1, l.e. medium is air, the focal length


Lens Maker's Formula for a of lens (i.e. fa) is given by
Convex Lens
This formula relates the focal length of a lens to
~ = (11-1) (~-~) ... (ii)
fa RJ R2
the refractive index oflens and radii of curvature
of two surfaces. (iii) fm = ((11
fa ~-1
-ll) [dividing Eq. (ii) by Eq. (i)]
.!. = (11 -1) (~ -~)
11'

R
f RJ R2
11 = refractive index of material of lens w.r.t. (iv) If 11 = 11' => fm = 00

surrounding media and RJ, R2 = radii of curvatures => lens behaves like a glass slab.
of two surfaces. (v) If 11 < 11' => fm> fa and nature reversed.
t;> fa

SI
A (vi) If 11 > 11' => and nature remains
same.

o
1.7 Linear Magnification
Produced by a Lens
Linear magnification of a lens is defined as, the
IT ratio of the height of the image formed by the lens
and height of the object.

Lin ear mag nificati


icanon, m Height of image (I)
= ---=-----=-"'-'-
Height of object (0)

For Convex Lens


H
CASE I When image is real, m = -I =~
o -u
When image is real, it is inverted and
forms on the other side of object.
O

CASE II When image is virtual, m = !... = ~


o u
When image is virtual, it is erect and
forms on the same side of object .
M

. (c) c Thus, it can be said convex lens gives


positive linear magnification for virtual
Some Important Points image and negative linear magnification
When lens of refractive index 11 is immersed in a for real image.
medium of refractive index 11', then
(i) When lens is taken in another medium, For Concave Lens
then focal length changes to fm which is Concave lens always forms virtual image, linear
given by magnification of concave lens,
I v
m=-=-.
o u
256 o ehapterwise eBSE Solved Papers PHYSICS

Concave lens always gives positive linear 1 1 1 1


-=-+-+ ...+-
magnification. Other formula for linear 111/2 In
magnification are
NOTE Magnification by combination of lenses
v
m=-=--=--
I -v I m = mi X m2 X m3 .
u I I+u
1.10 Prism
loB Thin Lens Formula

R
Prism have got the property of bending the
It is a relation between focal length of a lens and incident light towards its base.
distances of objects and image from optical centre A prism is a wedged shaped portion of a
of the lens. transparent medium bounded by two plane faces
1 inclined to each other at a suitable angle.

SI
• Lens formula =- - -
I v u We get, 1j +'2 = A and 0 = il + i2 - A
A Refracting edge

IT B'

I+---V
A'
I

• Lateral or transverse magnification, When the prism is adjusted at angle of minimum


I v I
m=-=-=--=--,
I-v deviation, then
o u I+u I (a) angle of incidence is equal to the angle of
H
emergence
where, angular magnification =~
8 (b) il = i2, 1j = '2' 0 = Om' A + Om = 2i and 2, = A.
where, 80 = angle made by image and 8= angle
made by object.
(c) ~ =Sin( A:O m
)/Sin~
O

• Hlml> 1 ::::} image is magnified. ::::}Om'" (u. -l)A


• Iflml < 1 ::::}image is diminished. (for small angle of prism)
• If Im I = 1 ::::}image is of same size as the object.
Dispersion by a Prism
M

The phenomenon of splitting of white light into


1.9 Power of Lens its component colours on passing through a
The ability of a lens to converge or diverge the rays refracting medium is known as dispersion.
of light incident on it is called the power of the
lens.

Thus, P = 1
I (in m)
White
light beam White light
S1 unit of power oflens = dioptre (D) = m-I spectrum
Power of combination lenses in contact is given by V
Glass prism
P =PI +P2 +...+Pn
CHAPTER 9 : Roy Optics And Opticol Instruments 257

Terms Related to Dispersion of Rayleigh Law of Scattering


It states that the intensity of light of wavelength A.
Light by Prism present in the scattered light is inversely
Angular Dispersion proportional to the fourth power of A., provided the
size of scattering particles are much smaller than A.,
Angular dispersion produced by a prism for
white light is the difference in the angles of Mathematically scattering [for a < < A.]. The
oc -;.
A.
deviation for two extreme colours i.e. violet and
bluishness of sky and reddishness of sunrise and
red.

R
sunset could be explained by this law.
Angular Deviations
For violet Ov = QJ.v-1)A
For red, OR = (Il R - I)A
and for yellow Oy = QJ.y -1)A
PREVIOUS YEARS'!

SI
Dispersive Power EXAMINATION QUESTIONS
Dispersive power of a prism is defined as the
ratio of angular dispersion to the mean
TOPIC 1
deviation produced by the prism.
Dispersive power, co = II
/""'"v
-II
/""'"R = 0-0
V R
o 1 Mark Questions
Ily -1 0 1. How does the angle of minimum deviation of
IT
Angular dispersion a glass prism vary, if the incident violet light
Mean deviation is replaced by red light? Give reason.
As A. red > A. violet All India 2017

.. Il red < Il violet 2. Why does sun appear red at sunrise and
Hence, Ored < Oviolet sunset? All India 2016
where, 0 = minimum
H
deviation. 3. Why does bluish colour predominate in a
Angular dispersion clear sky? Delhi 2015

= Ov -OR = (Ilv -1)A- (IlR -1)A 4. A concave lens of refractive index 1.5 is
= (Ilv -IlR)A immersed in a medium of refractive index
1.65. What is the nature of the lens?
O

= Ov - OR = (Ilv -IlR)A
All India 2015

Combining Two Thin Prisms


We can study two conditions
(i) Dispersion without average
H 0:/
A
5. When an object is placed between
of a concave mirror, would the image
f and 2f

formed be (i) real or virtual and (ii)


M

deviation . (0
diminished or magnified? Delhi 2015C
Ov - OR = QJ.y -1) A(co - co') A 6. A biconvex lens made of a transparent
(ii) Average deviation without dispersion material of refractive index 1.25 is
immersed in a water of refractive index
0= (Ily -l)A[I- :,] 1.33. Will the lens behave as a converging
or a diverging lens? Give reason.
All India 2014
Scattering of Light 7. A biconvex lens made of a transparent
Its a phenomena in which light is deflected from material of refractive index 1.5 is immersed,
its path due to its interaction with the particles in a water of refractive index 1.33. Will the
of the medium through which it passes. lens behave as a converging or a diverging
lens? Give reason. All India 2014
258 o Chopterwise C8SE Solved Papers PHYSICS

8. A convex lens is placed in contact with a 19. A lens behaves as a converging lens in air
plane mirror. A point object at a distance and a diverging lens in water (u = 4/ 3).
of 20 em on the axis of this combination What will be the condition on the value of
has its image coinciding with itself. refractive index <Il) of the material of the
What is the focal length of the lens? lens? Delhi 2011C
AllIndia2014
20. A converging lens axially in contact with a
9. Write the relationship between angle of diverging lens; both the lenses being of

R
incidence i, angle of prism A and angle of equal focal lengths. What is the focal
minimum deviations from a triangular length of the combination? AllIndia2010
prism. Deihl 2013 21. A glass lens of refractive index 1.45
10. When red light passing through a convex disappears when immersed in a liquid.

SI
lens is replaced by light of blue colour, What is the value of refractive index of the
40w will the focal length of the lens liquid? Deihl 2010
change? AllIndia2013C 22. Calculate the speed of light in a medium
11. How does focal length of a lens change whose critical angle is 30°. Deihl2010
when red light incident on it is replaced by
23. Why does the sky appear blue? Foreign 2010
violet light? Give reason for your answer.
Foreign 2012 24. Under what condition does the formation of
rainbow occur? AllIndia2010C
12. Name the physical quantity which
IT
remains same for microwaves of 25. Two thin lenses of power + 6D and - 2D
wavelength 1 mm and UV-radiation of are in contact. What is the focal length of
1600 A in vacuum. Deihl2012 the combination? AllIndia2010
13. Under what condition, does a biconvex 26. Two thin lenses of power + 4 D and - 2 D
lens of glass having a certain refractive are in contact. What is the focal length of
the combination? AllIndia2010
H
index act as a plane glass sheet when
immersed in a liquid? Deihl 2012 27. Two thin lenses of power + 5D and - 2.5 D
14. For the same value of angle of incidence, are in contact. What is the focal length of
the angles of refraction in three media A, the combination? AllIndia2010
Band Care 15°, 25° and 35° respectively. 28. Why are convex mirrors used as side view
O

In which medium, would the velocity of mirrors in cars? Delhi 2009C


light be minimum? AllIndia2012
15. When monochromatic light travels from o 2 Marks Questions
one medium to another, its wavelength 29. A ray PQ incident normally A
changes but frequency remains the same. p
M

on the refracting face BA is


Explain.Delhi 2011 refracted in the prism BAC Q
16. The refractive index of diamond is much made of material of
greater than that of glass. How does a refractive index 1.5.
diamond cutter make use of this fact? Complete the path of ray
HOTS; AllIndia2011C through the prism. From
17. If a ray of light propagatesfrom a rarer which face will the ray B
to a denser medium, how does its emerge? Justify your answer. AllIndia2016
frequency change? AllIndia2011C 30. Use the mirror equation to show that on
18. State the criteria for the phenomenon of object placed between F and 2F of a
concave mirror produces a real image
total internal reflection of light to take
beyond 2F. AllIndia2015
place. Delhi 2011. 2010. 200BC
CHAPTER 9 : Roy Optics And Optical Instruments 259

31. How does the refractive index of a (ii) Angle of minimum deviation,
transparent medium depend on the Dm = 2i - A Foreign 2014
wavelength of incident light used? Velocity
of light in glass is 2 x 108ml s and in air is
3 x 108ml s. If the ray of light passes from
glass to air, calculate the value of critical
angle. Foreign 2015 p

R
32. An equiconvex lens of focal length f is cut s
into two identical plane convex lenses. How
will the power of each part be related to the B C
focal length of the original lens? 36. Write the conditions for observing a
A double convex lens of + 5D is made of rainbow. Show by drawing suitable

SI
glass of refractive index 1.55 with both diagram how one understands the
faces of equal radii of curvature. Find the formation of a rainbow. All India 2014C
value of its radius of curvature. Foreign 2015 37. A ray PQ is incident A I

33. Two monochromatic rays of light are normally on the face P


incident normally on the face AB of an AB of a triangular
isosceles right-angled prism ABC. The prism of refracting
refractive indices of the glass prism for the angle of 60° made of
two rays 1 and 2 are respectively 1.35 and a transparent B 2 C
IT
1.45. Trace the path of these rays after material of refractive index ./.i as shown
in the figure. Trace the path"tJIthe ray as
entering through the prism.
it passes through the prism. Also,
A 450 calculate the angle of emergence and
angle of deviation. Delhl2014C
38. A convex lens of focal length it is kept in
H
contact with a concave lens of focal
B C All Indio 2014 length /2. Find the focal length of the
combination. All India 2013
34. A ray of light falls on a transparent sphere
39. When monochromatic light travels from
O

with centre C as shown in the figure. The


ray emerges from the sphere parallel to the a rarer to a denser medium, explain the
following giving reasons.
line AB. Find the angle of refraction of A if
the refractive index of material of sphere is (i) Is the frequency of reflected and
refracted light same as the
Ji Foreign 2014
frequency of incident light?
M

(ii) Does the decrease in speed imply a


reduction in the energy carried by
light wave? Deihl 2013
40. (i) Write the necessary conditions for
the phenomenon of total internal
reflection to occur.
35. Figure shows a ray of light passing through
a prism. If the refracted ray QR is parallel (ii) Write the relation between the
to the base BC, show that refractive index and critical angle
for a given pair of optical media.
(i) rl = r2 = A and Deihl 2013
. 2
260 o Chapterwise CBSE Solved Papers PHYSICS

41. A convex lens of focal length 25 em is 46. (i) Draw a ray diagram for a convex
placed coaxially in contact with a concave mirror showing the image formation
lens of focal length 20 cm. Determine the of an object placed anywhere in front
power of the combination. Will the system of the mirror.
be converging or diverging in nature? (ii) Use this ray diagram to obtain the
Delhi 2D13
expression for its linear
42. Trace the path of a ray of A magnification. All Indio 2012C

R
light passing through a
47. How does focal length of a lens change
glass prism ABC as
when red light incident on it is replaced
shown in the figure. If the
by violet light? Give, reason for your
refractive index of glass is
r{ ./3, then find out the answer. Foreign 2D12

SI
value of the angle of 48. Two thin lenses of power - 4 D and 2 D
fIR I mergence from the B C are placed in contact coaxially. Find the
prism. HDTS; Foreign 2D12 focal length of the combination. All India 2012C
4~. A ray of light incident on an equilateral 49. Draw a ray diagram to show the image
glass prism (/J. g = ./3) moves parallel to formation by a concave mirror.

the base line of the prism inside it. Find When the object is kept between its focus
the angle of incidence for this ray. Deihl 2012 and the pole.
IT
U sing this diagram, derive the
44. An object AB is kept in front of a concave
magnification formula for the image
mirror as shown in the figure.
formed. Delhi 2011
A 50. A beam of light converges at a point P. A
concave lens of focal length 16 em is
placed in the path of this beam 12 em from
H
B C F P. Draw a ray diagram and find the
location of the point at which the beam
would now converge. All India 2011C
51. Draw a diagram showing the formation of
O

(i) Complete the ray diagram showing primary rainbow and explain at what
the image formation of the object. angles the primary rainbow is visible.
(ii) How will the position and intensity of Delhi 2010C
the image be affected if the lower _ 52. The radii of curvature of the faces of a
half of the mirror's reflecting surface double convex lens are lOcm and 15 ern. If
M

is painted black? All Indio 2012 focal length of the lens is 12 em, find the
45. (i) Plane and convex mirrors are known refractive index of the material of the
to produce virtual images of the lens. Delhi 2010
objects. Draw a ray diagram to show 53. (i) The bluish colour predominates in
how, in the case of convex mirrors, clear sky.
virtual objects can produce real (ii) Violet colour is seen at the bottom of
images. the spectrum when white light is
(ii) Why are convex mirrors used as side dispersed by a prism. State reasons
- view mirrors in vehicles? to explain these observations.
Delhi 2012C Delhi 2010
CHAPTER 9 : Roy Optics And Optical Instruments 261

54. A biconvex lens has a focal length 2/3


times the radius of curvature of either
o 3 Marks Questions
surface. Calculate the refractive index of 61. (i) Monochromatic light of wavelength
lens material. Delhi 2D10 589 nm is incident from air on a
water surface. If u for water is 1.33,
55. (i) Why does the sun appear reddish at find the wavelength, frequency and
sunset or sunrise?
speed of the refracted light.
(ii) For which colour, the refractive (ii) A double convex lens is made of a
index of prism material is maximum glass of refractive index 1.55 with

R
and minimum? Delhi 2010 both faces of the same radius of
56. Find the radius of curvature of the convex curvature. Find the radius of
surface of a plano-convex lens, whose focal curvature required, if the focal length
length is 0.3 m and the refractive index of is 20 cm. All India 2017
the material of the lens is 1.5. Deihl 2010

SI
62. (i) A ray of light incident of face AB of an
57. (i) Out of blue and red light, which is equilateral glass prism, shows
deviated more by a prism? Give minimum deviation of 30°. Calculate
reason. the speed of light through the prism.
(ii) Give the formula that can be used to
determine refractive index of
material of a prism in minimum
deviation condition. Deihl 2010 aLe
IT
58. The following table gives the values of the (ii) Find the angle of incidence at face
angle of deviation for different values of AB, so that the emergent ray grazes
the angle of incidence for a triangular along the face AC. Delhi 2017
prism.
63. (i) Calculate the distance of an object of
Angle of 33° 38° 4r 52° 60° 71° height h from a concave mirror of
radius of curvature 20cm, so as to
H
incidence
Angle of 60° 50° 46° 40° 43° 50°
obtain a real image of magnification
deviation 2. Find the location of image also.
(ii) Using mirror formula, explain why
(i) For what value of the angle of does a convex mirror always produce
incidence, is the angle of emergence
O

a virtual image? Delhi 2016


likely to be equal to the angle of
incidence itself?
64. In the following diagram, an object '0' is
placed 15cm in front of a convex lens ~ of
(ii) Draw a ray diagram showing the
focal length 20 em and the final image is
passage of a ray of light through this
formed at I at a distance of 80 cm from the
prism when the angle of incidence
M

second lens L2. Find the focal length of the


has the above value. Delhi 2010C
lens L2•
59. (i) State the principle on which the
20 em so em
working of an optical fibre is based.
(ii) What are the necessary conditions for
this phenomenon to occur? All India 2009
....-
15em

60. (i) What is the relation between critical


angle and refractive index of a
material?
(ii) Does critical angle depend on the
colour of light? Explain. All India 2009
262 o Chapterwise CSSE Solved Papers PHYSICS

65. (i) A mobile phone lies along the 70. You are given three lenses ~ , L2 and La
principal axis of a concave mirror. each offocallength 10 cm.An objectis kept at
Show with the help of a suitable 15 em in front of ~ as shown. The final real
diagram, the formation of its image. image is formed at the focusI of La.
Explain why magnification is not a Find the separation between ~ , L2 and La. All
uniform. India 2012
(ii) Suppose the lower half of the
concave mirror's reflecting surface

R
is covered with an opaque material.
What effect this will have on the ~----~+----4-4----~-+----I
image of the object? Explain.
Delhl2D14
71. A small bulb (assumed to be a point source)

SI
66. A convex lens of focal length 20 em is
is placed at the bottom of a tank containing
placed coaxially with a convex mirror of
water to a depth of 80 cm. Find out the area
radius of curvature 20 cm. The two are
of the surface of water through which light
kept at 15 em from each other. A point
from the bulb can emerge. Take the value of
object lies 60 em in front of the convex
the refractive index of water to be 4/3.
lens. Draw a ray diagram to show the
Delhl2013C
formation of the image by the
combination. Determine the nature and 72. Draw a ray diagram to show the formation
IT of the image of an object placed on the axis
position of the image formed. All India 2014
of a convex refracting surface of radius of
67. A convex lens of focal length 20 cm is
curvature 'R', separating the two media of
placed coaxially with a concave mirror of
refractive indices 'ni' and 'n2' (n2> ni).
focal length 10 em at a distance of 50 cm
Use this diagram to deduce the relation
apart from each other. A beam of light
coming parallel to the principal axis is n2 _ ni = n2 - ni, where u and v represent
H
v u R
incident on the convex lens. Find the
respectively the distance of the object and
position of the final image formed by
the image formed. Deihl 2012 C
this combination. Draw the ray diagram
showing the formation of the image. 73. A convex lens made up of a glass of
refractive index 1.5 is dipped in turn in
O

All India 2014


68. Define the term 'critical angle' for a pair (i) a medium of refractive index 1.65
of media. (ii) a medium of refractive index 1.33
(a) Will it behave as a converging lens or
A point source of monochromatic light '8' a diverging lens in the two cases?
is kept at the centre of the bottom of a (b) How will its focal length change in the
M

cylinder of radius 15.0 cm. The cylinder two media? All India 2011
contains water (refractive index 4/3) to a
74. Use the mirror equation to show that
height of 7.0 em. Draw the ray diagram
and calculate the area of water surface (i) An object placed between f and 2f of a
through which the light emerges in air. concave mirror produces a real image
beyond 2f.
Delhl2013C
(ii) A convex mirror always produces a
69. Define power of a lens. Write its units. virtual image independent of the
Deduce the relation.!. = ~ + ~ for two location of the object.
f it f2 (iii) An object placed between the pole and
thin lenses kept in contact coaxially. focus of a concave mirror produces a
Farelgn 2012 virtual and enlarged image. All India 2011
CHAPTER 9 : Roy Optics And Optical Instruments 263

75. Find the position of the image formed of 82. An illuminated object and a screen are
the object 0 by the lens combination given placed 90 em apart. Determine the focal
in the figure. Foreign 2011 length and nature of the lens required to
f= +10cm-10cm 30cm produce a clear image on the screen, twice
the size of the object. All India 2010

o 83. (i) How is the focal length of a spherical
mirror affected when the wavelength

R
1-30cm
of the light used is increased? '
(ii) A convex lens has 20 em focal length
I 5cm I 10cm I in air. What is its focal length in
76. A converging lens has a focal length of water? (Refractive index of air-water
20 em in air. It is made of a material of = 1.33, refractive index of air-glass

SI
refractive index 1.6. It is immersed in a = 15). Foreign 2010

liquid of refractive index 1.3. Calculate its 84. (i) How is the focal length of a spherical
new focal length. All Indio 2011 mirror affected when it is immersed
77. State the necessary conditions for in water?
producing total internal reflection of light. (ii) A convex lens has 10 em focal length
Draw ray diagrams to show how specially in air. What is its focal length in
designed prisms make use of total water? (Refractive index of air-water
internal reflection to obtain inverted = 1. 33, refractive index of air-glass
IT
image of the object by deviation of rays = 15). Foreign 2010
(i) through 90° and 85. An object of 3 em height is placed at a
(ii) through 180°. All Indio 2011 distance of 60 cm from a convex mirror of
78. With the help of a suitable ray diagram, focal length 30 cm. Find the nature,
derive a relation between the object- position and size of the image formed.
H
distance (u), image distance (v) and radius All India 2010C

of curvature R for the convex spherical 86. An object of 2 em height is placed at a


surface when a ray of light travels from a distance of 30 em from a convex mirror of
rarer to denser medium. OelhI2011C, 2008 C focal length 15 cm. Find the nature,
79. A ray of light is incident on one face of a position and size ofthe image formed.
O

glass prism and emerges out from the All India 2010C
other face. Trace the path of the ray and 87. Three light rays, A
derive an expression for refractive index red (R), green (G)
of the glass prism. Deihl 2011C and blue (B) are B
80. The image obtained with a convex lens is incident on a G
M

erect and its length is four times the R


right angled
length of the object. prism ABC at
If the focal length of the lens is 20 em, face AB. The B'----, ----'---"'C
calculate the object and image distances. refractive indices of the material of the
All India 2010 prism for red, green and blue wavelengths
81. A convex lens is used to obtain a are 1.39, 1.44 and 1.47 respectively. Out
magnified image of an object on a screen of the three, which colour of ray will
10 em from the lens. If the magnification emerge out of face AC ? Justify your
is 19, find the focal length of the lens. answer. Trace the path of these rays after
All India 2010 passing through face AB. HOTS, Dllhl 2009
o Chapterwise CBSE Solved Papers PHYSICS

88. State the conditions under which total


internal reflection occurs. One face of a
o 5 Marks Questions
prism with a refracting angle of 30° is 92. (i) Derive the Mathematical relation
coated with silver. A ray incident on between refractive indices n1 and n2
another face at an angle of 45° is refracted of two radii and radius of curvature
and reflected from the silver coated face R for refraction at a convex

R
and retraces its path. Find the refractive spherical surface. Consider the
index of the material of the prism. object to be a point since lying on
Foreign 2009 the principal axis in rarer medium
89. A tank is filled with water to a height of of refractive index nl and a real
12.4 cm. The apparent depth of a needle image formed in the denser medium

SI
lying at the bottom of the tank is measured of refractive index n2' Hence, derive
by a microscope to be 9.3 cm. What is the lens maker's formula.
refractive index of water? If water is (ii) Light from a point source in air falls
replaced by a liquid of refractive index 1.6 on a convex spherical glass surface
up to the same height, by what distance of refractive index 1.5 and radius of
would the microscope have to be moved to curvature 20 cm. The distance of
focus on the needle again? light source from the glass surface
is 100 cm. At what position is the
Delhi 2009C
IT
image formed. All India 2016
90. When a ray of light passes through a
triangular glass prism, find out the relation 93. (i) Plot a graph to show variation of
for the total deviation, 0 in terms of the the angle of deviation as a function
angle of incidence, i and angle of of angle of incidence for light
emergence, e. Plot a graph showing the passing through a prism. Derive an
H
variation of angle of deviation with the expression for refractive index of
angle of incidence and obtain the condition the prism in terms of angle of
for the angle of minimum deviation. minimum deviation and angle of
All Indio 2009C
prism.
(ii) What is dispersion of light? What is
91. Light incident at an angle of incidence of
O

its cause?
45° in a certain medium goes grazing along
its surface of separation from air after (iii) A ray of light incident normally on
refraction. one face of a right isosceles prism is
totally reflected as shown in figure.
What is What must be the minimum value
M

(i) the velocity of light in this medium? of refractive index of glass? Give
(ii) the angle of incidence at which light relevant calculations.
from air must be incident on this
medium so that the refracted ray is
normal to the reflected ray? Also,
name the special property exhibited by
light for this angle of incidence.
All India 2015
All India 2009C
CHAPTER 9 : Roy Optics And Optical Instruments 261

94. (i) A point object 0 is


kept in a medium
n1 (iE2
----O_--u-_- ---R---~--
96. (i) A point object is placed in front of a
double convex lens (or refractive
of refractive index index 11 = n2 with respect to air)
n1 in front of a n1
convex spherical surface of radius of with its spherical faces of radii of
curvature R which separates the curvature ~ and R2• Show the path
second medium of refractive index n2 of rays due to refraction at first and
from the first one, as shown in the

R
subsequently at the second surface
figure. to obtain the formation of the real
Draw the ray diagram showing the image of the object.
image formation and deduce the
Hence, obtain the lens maker's
relationship between the object
formula for a thin lens.
distance and the image distance in

SI
terms of n1, n2 and R. (ii) A double convex lens having both
faces of the same radius of
(ii) When the image formed above acts as a
curvature has refractive index 1.55.
virtual object for a concave spherical
Find out the radius of curvature of
surface separating the medium n2 from
the lens required to get the focal
n1(n2> n1), draw this ray diagram and
length of 20 cm.
write the similar [similar to (i)]
All India 2014 C
relation. Hence, obtain the expression
for the lens maker's formula. 97. Draw a ray diagram showing the
IT
All India 2015 formation of the image by a point object
95. (i) A ray PQ of light is incident on the face
on the principal axis of a spherical
ABof a glass
convex surface separating two media of
prism ABC (as A refractive indices n1 and n2' when a
shown in the point source is kept in rarer medium of
figure) and refractive index n1. Derive the relation
H
emerges out of between object and image distance in
the face AC. terms of refractive index of the medium
Trace the path of and radius of curvature of the surface.
the ray. Show that Zi + Le = LA + Lo Hence, obtain the expression for lens
where, 0 and e denote the angle of maker's formula in the case of thin
O

deviation and angle of emergence convex lens. Delhi 2014C


respectively. 98. (i) Draw a ray diagram to show
Plot a graph showing the variation of refraction of a ray of
the angle of deviation as a function of monochromatic light passing
angle of incidence. State the condition through a glass prism. Deduce the
M

under which Lo is minimum. expression for the refractive index


(ii) Find out the relation between the of glass in terms of angle of prism
refractive index (11) of the glass prism and angle of minimum deviation.
and LA for the case, when the angle of (ii) Explain briefly how the
prism (A) is equal to the-angle of phenomenon of total internal
minimum deviation (om)' Hence, obtain reflection is used in fibre optics.
the value of the refractive index for Oelhi 2011
angle of prism A = 60°. Delhi 2015
266 o Chapterwise CSSE Solved Papers PHYSICS

99. (i) Obtain lens maker's formula using the curvature from the central point of the
expression spherical surface. Hence, derive the
~ _ n1 = (~ - n1) expression of the lens maker's formula.
v u R Delhi 2009

Here, the ray of light propagating 102. (i) Draw a ray diagram for formation of
from a rarer medium of refractive image of a point object by a thin
index (n1) to a denser medium of double convex lens having radii of
refractive index (n2) is incident on the

R
curvatures Rt and R2 and hence,
convex side of spherical refracting derive lens maker's formula.
surface of radius of curvature R.
(ii) Define power of a lens and give its
(ii) Draw a ray diagram to show that SI unit. If a convex lens of length
image formation by a concave mirror 50 em is placed in contact coaxially

SI
when the object is kept between its
with a concave lens of focal length
focus and the pole. Using this
20 em, what is the power of the
diagram, derive the magnification
formula for the image formed. Delhl2D11 combination? Foreign 2009
103. (i) Draw the ray diagram for the
100. Figure shows a convex spherical surface
with centre of curvature C separating the formation of image of an object by a
convex mirror and use it (along with
two media of refractive indices nl and n2.
Draw a ray diagram showing the formation the sign convention) to derive the
mirror formula.
IT
of the image of a point object 0 lying on the
principal axis. Derive the relationship (ii) Use the mirror formula to show that
between the object and image distance in for an object kept between the pole
terms of refractive indices of the media and and focus of a concave mirror the
the radius of curvature R of the surface. image appears to be formed behind
Delhi 2011 the mirror. All India 2009C
H
o
n1
o Explanations
1. Wavelength of violet light is smaller than that of
O

red light. Also, angle of minimum deviation,


om =(J.J. -l)A =>0m oc Il
As, IlR < Ilv =>(Om)R < (Om)V
B
As, deviation is less for red light, hence, angle of
G deviation decreases. rn
M

R 2. During sunrise and sunset, the rays have to


travel a larger part of the atmosphere because
45°
b L-----.l..-~c they are very close to the horizon. According to
Rayleigh's law of scattering, scattering oc ~,
101. Trace the rays of light showing the A
formation of an image due to a point object wavelength of red is large, hence it is least
placed on the axis of a spherical surface scattered.
separating the two media of refractive Therefore, light rays other than red is mostly
indices n1 and n2. Establish the relation scattered away. Most of the red light, which is
the least scattered, enters our eyes. Hence, the
between the distance of the object, the
sun appears red at sunrise and sunset. (1l
distance of image and the radius of
CHAPTER 9 Roy Optics And Optical Instruments 267

3. The blue colour of the sky is due to the scattering form an image coinciding with the object itself.
of sunlight by the molecules of the atmosphere. So, the image is formed at the focus of the lens.
According to Rayleigh's law of scattering, the So, focal length of the lens is 20 cm.
intensity of scattered light, I ec -;., blue colour
A L M
having the short wavelength in the visible
spectrum scatter the most. When we look at the
sky, the scattered light enters our eyes and this

R
light contains blue colour in a large proportion, so
the sky appears blue. (1)

4. A concave lens is made up of certain material


(1/2)
behaves as a diverging lens, when it is placed in a
medium of refractive index less than the 9. The relation between the angle of incidence i,
angle of prism A and the angle of minimum

SI
refractive index of the material of the lens and
behave .s a converging lens, when it is placed in deviation, ~m for a triangular prism is given by
a mcc.um of refractive index greater than the i = A + ~m.
r, ,-.cnve index of the material of the lens. (1) 2 m
In the given case, concave lens is immersed in 10. Focal length of the lens decrease when red light is
medium having refractive index greater than the replaced by blue light. (1)
refractive index of the material of the lens
(1· 65> 1· 5). Therefore, it will behave as a 11. This question can be answered by considering the
converging lens. lens maker's formula.
IT From the formula, we can identify which factor will
5. When an object is placed between / and 2/ of a
change on changing the wavelength.
concave mirror, the image formed is real. inverted
and magnified. The refractive index of the material of a lens
increases with the decrease in wavelength of the
incident light. So, focal length will decrease with
decrease in wavelength according to the formula.
H
~ = (J.I. -1) (~ - ~)
2F ,
/ R\ R2
6. When a lens is placed in a liquid, where rei ctive
index is more than that of the material of lens, Thus, when we replace red light with violet light
then the nature of the lens changes. So, when a then due to increase in wavelength the focal
O

biconvex lens of refractive index 1.25 is immersed length of the lens will decrease. (1)
in water (refractive index 1.33), i.e. in the liquid 12. Both microwave and UV-rays are a part of the
of higher refractive index, its nature will change. electromagnetic spectrum. Thus, the physical
So, biconvex lens will act as biconcave lens or quantity that remains for both types of radiation
diverging lens. (1) will be their speeds equal to c.
7. A biconvex lens acts as a converging lens in air c = 3xl08 m/s
M

(1)
because the refractive index of air is less than
that of the material of the lens. The refractive 13. When refractive index of lens is equal to the
index of water is less than the refractive index of refractive index of liquid. (1)

the material of the lens (1.5). So, its nature will r sin i c
14. From Snell slaw, J.I. = -- =-
not change, it behaves as a converging lens. (1) sin r v
8. The adjacent figure shows a convex lens L in ~ v cc sin r for given value of i
contact with a plane mirror M. P is the point ~ Smaller angle of refraction, smaller the
object kept in the front of this combination at a velocity of light in medium.
distance of 20 em from it. (112) Velocity of light is minimum in medium A as angle
Since, the image is coinciding with the object of refraction is minimum, i.e. 15°. (1)
itself, the rays from the object after refraction
from the lens fall normally on the mirror M md
268 o Chopterwlse eBSE Solved Papers PHYSICS

15. Because refractive index for a given pair of media 24. Availability of rain drops causes refraction,
depends on the ratio of wavelengths and velocity dispersion and total internal reflection of sun
of light in two medium and not on frequency. (1) light results in the form of rainbow and the back
16. The refractive index of diamond is much higher of the observer should be towards the sun. (1)

than that of glass. Due to high refractive index, the 25. Resultant power of the combination,
critical angle for diamond air interface is low. The
p = ~ + P2 = 6 - 2 = 4D
diamond is cut suitably so that the light entering
the diamond from any face suffers multiple total -1 = 4 ~ / = -1 m = 25 em

R
internal reflections at the various surfaces. (1) / 4 (1)

17. Frequency remains unchanged when light travels 26. Refer to Ans. 25, (Ans. / = 50 em) . (1)
from one transparent medium to another
transparent medium. (1) 27. Refer to Ans. 25, (Ans.f = 40 em) . (1)

18. Following are the criteria for total internal 28. Because convex mirror forms virtual. erect and

SI
reflection smaller image of object irrespective of relative
(i) Light must pass from a optically denser to a position of object from mirror and therefore, its
optically rarer medium. field of view is very wide. '(1)
(ii) Angle of incidence in denser medium is must be
29. Given, refractive index of A
greater than critical angle for two media. (1)
the material of the prism,
Il = 1.5 p
19. When a lens is immersed in a liquid whose refractive
index is more than that of the material of lens, then :. Critical angle for the
nature of lens changes, i.e. converging lens behaves material
like diverging lens and vice-versa. sin C = .!.. = _1_ = 2/3
IT
Il 1.5
Refractive index of the material of lens is less than
the refractive index of water. (1) ~ C = sin'? (~) =4~.
8L.....l.=----JC
20. Combined focal length of a lens combination
From the ray diagram, it
.!.. = 2.. + 2.. (For two thin lenses in contact) is clear that angle of incidence i = 30°< C.
1 It 12
H
Therefore the ray incident at the face AC will not
As, /2 = - It suffer total internal reflection and merges out
(focal lengths are equal, one is convex and other is through this face. (2)
concave)
30. According to the mirror equation, we have
1
~ -= 0 ~ /=00. 1 1 1
1 rn -+-=-
O

V u /
21. When a lens immersed in a liquid disappears then,
Illiquid = Il 9 = 1.45 (1)
where, u = distance of the object from the mirror
v = distance of the image from the mirror
22. Critical angle is the angle of incidence for which / = focal length of the mirror
angle of refraction becomes 90°. Here, in this case
M

. . d 1
ref ractive In ex, 1.1= -,-,-
From the mirror equation, we have
sin 'c v=~u- / ...(i)
lve ind
.: Rfe racnve m ex, Il = -c = -.-.-1 Applying new cartesian sign convention, we get
v sm1c
/ =- ve and u ve =-
~ v= c sin ic = 3xl08 x sin300 Given, / < u < 2/
= 3X108 x ~ = 1.5 X 108 m/s
~ v=-ve [fromEq.(i))
2 (1) Magnification is given by
23. Due to large scattering of visible light of smaller
wavelength (blue colour) as intensity of scattered m=-(=:)=-ve
light oc-;.~ rn Hence, the image formed is real. (1)
CHAPTER 9 : Ray Optics And Optical Instruments 269

From the mirror formula, when u = - 2/. (ii) From lens maker formula, we have
1 1 1
~ --+-=-
-2/ v -/
p = (1l-1 )(2.- -~)
Rl R2
1 1 -1
~ -=---=-
v 2/ / 2/ 5= ~.55-1)H-CR)}
Therefore, when the object is at [, then image is
formed at infinity. Shows that when 5= 0.55X2
R
/ < u < 2/, 00< v < 2/

R
(1)

31. (i) The refractive index of a transparent medium R = 0.55 x 2 = 0.22m = 22cm
5 (1)
is inversely proportional to the wavelength of
incident light. The relationship between the 33. The paths are shown as below:
two is given by
A

SI
Il=~
A (1)
where,
Il = Refractive index of medium
AO = Wavelength of incident light in vacuum
A = Wavelength of incident light in medium 45°
(ii) Given,
Velocity of light in air, va = 3xl08m/s
IT 2'
Velocity of light in glass, V9 = 2xl08 m/s
From the figure, it is clear that angle of incidence
The refractive index of glass is given by, for ray 1 is 4SO.
119 = .£. Where c is speed of light in vacuum. For ray 1,
v9
sin i = sin 450 = 1
.J2 1.414
The refractive index of air is given by, Ila = !2 For ray 1, the refractive index of the prism is
H
va
Il = 1.35
:. The refractive index of glass W.r.t. air will be
a _1l9 a _Va _3X102x1015 8
_
Il = -- 1 ~ .
sme = -1 = --
1
sine Il 1.35
119 - ~ ~ 119 -;- - 8 - .
"'a 9 Here, _1_ < 1
1.414 1.35
O

We know a 1 (1)
119 = sine i.e. sin i < sin e or i < e
where, c is the critical angle for the interface So, ray 1 will be refracted by the prism.
:. _1_ = 1.5 ~ sine = 2.- For ray 2, angle of incidence, i = 45°
sine 1.5
sin i = sin 4SO= ~ = _1_
~ e = sin " (0.66) ==) c = 41.3° .J2 1.414
M

:. Critical angle c = 41.3° (1) For ray 2, the refractive index, Il = 1.45
32. (i) The focal length of original equiconvex lens is 1
Il=-
f. sine
. 1 1
Let the focal length of each part after cutting ==) sme = - = --
beF. Il 1.45
1 1 1 1 2 F Here, _1_ > _1_
Here, - = - + - ~ - =- ~ / = - ==) F = 2/ 1.414 1.45
/ F F / F 2
i.e. sini> sine or i » e
Power of each part will be given by
So, ray 2 will get total internally reflected. (1)
P=.!..==)P=~
F ~ m
270 o ehapterwise eSSE Solved Papers PHYSICS

34. Given, i = 60 0
, ~ = .f3
From Snell's law, we have
sini
-=~
sinr
sin60° =.f3
sinr
. sin 60 0
1 .f3
smr=--=-x-
.f3 .f3 2

R
SI
sinr = 0.5
r = sin'? (0.5) (bJ Secondary Rainbow

r = 30 0
(1)

35. (i) When QR is parallel to the base BC, we have


1:
~-------+--...p.~:.-.....
i = e (prism is in the position of minimum
deviation)
IT c::
A ~ -_----+-+---+-+-+-~- R' Primary
V"

(1)
H
p
s
BL...L-----~, Formation of rainbow
(1)
The rays of light reach the observer through a
~ r1=r2=r (let) ... (i)
O

refraction followed by a reflection, followed by a


We know that
refraction.
r1+r2=A
Figure shows red light from drop 1 and violet
From Eq. (i), we get
light from drop 2, reaching the observer eye.
2r = A. r = ~ ... (ii)
2 37. If the i, is the critical angle for the prism.
M

A 1
rl=r2=- Now, 11 =--
2 sinie
(ii) Also, we have
.. 1 1 .f3
A+D=i+e .. sin Ie =- =- =-
Substituting, D = Dm and e = i 11 2 2
.f3
A+ Dm = i + i
.. Dm = 2i - A (1) ~ . i, = sin'" ( ~) ~ ie = 600

36. The conditions for observing a rainbow are given


below: Angle of incidence at face AC of the prism is 600•
(i) The sun comes out after a rainfall or shower. Hence, refracted ray grazes the surface AC. Angle
of emergence = 900
(ii) The observer stands with his/her back towards
the sun. Angle of deviation = 300 (2)
CHAPTER 9 : Roy Optics And Opticol Instruments 271

38. Focal length for convex lens = fi At the interface AC, we have (according to Snell's
Focal length for concave lens = - 12 law)
The equivalent focal length of a combination of A
convex lens and concave lens is given by
~=~+_1_ ~ F= fil2
F fi - 12 12 - fi (2)

39. (i) The frequency of reflected and refracted light

R
remains same as the frequency of incident
light because frequency only depends on the
source of light. (1)
(ii) Since, the frequency remains same, hence
there is no reduction in energy. (1)

SI
40. (i) Refer to ADs. 18. (1)

(ii) = _1_, where a and b are the rarer and


al1b sini=119
sin C
sin r l1a
denser media, respectively. C is the critical
angle for the given pair of optical media. (1) But, sin i = sin 0° = 0
41. Given, focal length of convex lens, Thus, sin r = l1asin i = 0
119
fi = + 25cm = + 0.25 m and focal length of
concave lens, 12 = - 20 em = - 0.20 m Hence, r=0
IT
Equivalent focal length of convex and concave This ray pass unrefracted at AC interface and
lens, reaches AB interface. Here, we can see angle of
F=~+ ~=~+ _1_= __ 1_ incidence becomes 30°.
fi 12 25 - 20 100 Thus, applying Snell's law
.. F=-lOOcm=-lm
sin 30° _ l1a _ 1
sine - 119- ,J3
Now, the power of lens, P = ~
H
I sin e = ,J3 x sin 300= ,J3
1 1
For convex lens, P, = - = - 2
fi 0.25 Thus, e = 60°
1 1 Hence, angle of emergence is 60°. (1)
For concave lens, P2 = - = --
12 -0.20
O

Hence, power of the combination 43. To draw the ray diagram for the refraction from
the prism. FOllowing things should be kept in the
P = P, + P = _1_ + _1_ = 100 + 100 mind.
2 0.25 - 0.20 25 - 20
(i) Draw normal to the point of incidence.
= 400 - 500 = - 100 = _ I D
(ii) Consider each boundary of the prism as separate
100 100
M

interface and draw the ray diagram for the


Here, the focal length of the combination refraction taking place.
=100 cm= -I m
The reflection of light through prism is shown as
Since, the focal length is in negative, so the system below:
will be diverging in nature. (2)
A
42. While tracing the path of the ray, we should
remember that prism bends the incident rays
towards its base.

Refractive index of glass, 119= ,J3 (1)


Since, i = 0
272 IZI ehopterwise eBSE Solved Papers PHYSICS

By geometry Angle of refraction, r = 30° (112) 46. (i)


Given, refractive index, Ii = .J3
Using Snell's law, Ii = sin j
sin r (112)
=> sin j = Ii sin r c
= (.J3) sin (300) = .J3
2 (1/2)
v

R
Angle of incidence, { = 60° = ~
3 Figure shows the formation of image A' B' of a
. 1t finite object AB by a convex mirror, virtual,
z=- (1/2)
erect and diminished. (1)
3
(ii) Now, MBP -M'B'P
44. To draw the ray diagram for image formation, the A'B' PB'

SI
:.--=-
following are the rules to form the image from AB PB
spherical mirror.
Applying the new cartesian sign convention,
(i) The ray parallel to principal axis passes through A'B'= h» AB = hI' PB'= v, PB =-u
the focus after reflection. .. h2 =.!.....
(ii) The ray passing through the focus becomes hI -u
parallel to principal axis after reflection.
Linear magnification m = h2 = -~
(iii) The ray passing through the centre of curvature hI u (1)
returns on the same path after reflection.
47. We know.!.. = (IJ.-l)(~-~)
IT
(i) The ray diagram showing the image f RI R2
formation of the object (1) 1
f oc -- and liv > IiR
(IJ.-1)
The increase in refractive index would result in
decrease of focal length of lens. Hence, we can say
that replacing red light with violet light, decreases
H
the focal length of the lens used.
AB = Object 48. Net power P = 1l. + P2 = -4+ 2=-2D
A'B' = Image 1 1
Focal length f = - = -m = -O.5m = -SOcm
P -2
(ii) The position of the image remains same
O

whereas intensity of image reduces. 49. Ray diagram of image formation by a concave
(112+1/2 = 1) mirror. M"
45. (i) If a plane or a convex mirror is placed in the
path of rays converging to a point the rays get
reflected to a point in front of the mirror. p
c N"
M

Real image can be obtained on a screen. (1)


Real
image Virtual
object .• 0
In t,M'N'P and M" N"P
...>:::.::>:" M" N"
------
N" P
M'N' N'P
By sign convention, PN' = - u, PN" = +v
M'N' = hI and M"N" = h2
h2=~
hI - U
(ii) The convex mirror is used as side view
:.Linear magnification, m = h2
v
mirrors in vehicles as it gives a wide field of m=--
view of the traffic. (1) hI u (2)
CHAPTER 9 : Roy Optics And Optical Instruments 273

50. Ray diagram 54. Given, f = 3 R, R


J = + R, R2 = - R
3
Using Lens maker's formula,
2.. = (Il - 1) (2.. - 2..)
f RJ R2
~ = (Il -1) (2.. + 2..)
2R R R
m

R
3

~ = (1l-1) (i)
1l-1=~
4

SI
3 7
Il = 1 + - ~ Il = - = 1.75 (1)
(1) 4 4
Given, U =+ 12 cm , f =- 16 cm , v =? 55. (i) Reddishness at sunset and sunrise When
Using lens equation, the sun is near the horizon at sunset or
1 1 1 1 1 1 sunrise, the light rays have to traverse a larger
-=--- ~ --=--- thickness of the atmosphere than when the
f v U 16 v 12
sun is overhead at noon. In accordance with
~ 1 1 _=4-3=2.. Rayleigh's scattering law, the lower
v 12 16 48 48 wavelengths in the blue region are almost
v=+48cm
IT completely scattered away by the air
The image of virtual object at P forms at P' at a molecules.The higher wavelengths in the red
distance 48 cm from the lens. (1)
region are least scattered and reach our eyes.
Hence, the sun appears almost reddish at
51. For ray diagram for the formation of rainbow sunset and sunrise. (1)
Refer to Ans. 36. (1)
(ii) Refractive index Il of prism is maximum for
The primary rainbow is formed by those rays violet and minimum for red colour. (1)
H
which suffer one total internal reflection and two
refraction and comes out of the rain drop at angle 56. For a plano-convex lens, RJ = 00

of minimum deviation. R2 = - R; f = 0.3 m= 30 cm


The violet and red light colours emerge as cone of Il = 1.5
rays at 41 and 43 respectively and can be viewed
0 0

R=?
by observer. (1)
O

Lens maker's formula,


f =+
52. Given, RJ =+ 10 ern R2=-15cm,
Lens maker's formula,
12cm, Il =?
(1)
2.. = (Il -1) (2.. - 2..)
f
2.. = (Il -1) (2.. - 2..) RJ R2 (1)

f RJ R2 ~ 2.. = (Il -1) (~ __ 1_)


(2) 30 R
M

00 -

~ ~ = (Il -1) (~ + ~) = (Il -1)


12 10 15 30 2..=(1.5-1)
1 30 R
~ 1l-1=-
2 ~ R=15cm (1)
3 57. (i) Blue light suffers more deviation by a prism
1l="2
(1) than red light.
53. (i) Refer to Ans. 3. (1) This happens due to high value of refractive
(ii) Violet colour can be seen at bottom as it index of material of prism for blue light because
undergoes largest deviation in the visible of its smaller wavelength in visible spectrum(1)
spectrum because of highest value of (ii) om = (Il -1),1. where om = angle of minimum
refractive index, Il of prism for this colour deviation
being of smaller wavelength. (1) Il = refractive index, A = prism angle. (1)
274 IZI ehapterwise eSSE Solved Papers PHYSICS

58. (i) i = 52°. When prism is adjusted at an angle of VI 3 X 108 8-1


=> v2 = - = --- = 225 x 10 ms
minimum deviation, then angle of inddence is 1i21 1.33
equal to the angle of emergence because
i + e = A + o. (1)
.. A2 = ~ = 589 = 442.85 = 443nrn
1i21 1.33
(ii) The ray diagram in the condition of minimum
deviation is shown as below: So, wavelength of reflected beam = 443 nrn and
its speed = 2.25 x 108 ms'"
A
(ii) For a biconvex lens, using lens maker s formula,

R
.!. =/J! -1) (..!..-..!..)
I RI R2
Here, 1=20 em, Ii = 1.55 => RI = + R and
R2 =-R
s 1 2

SI
:. We have, - =/J! -1)-
p I R
=> R = 2(1i -1)1 = 2 x (1.55-1) x 20= 22cm
:. Radius of 22 em is required.
om
Angleo!
t 62. (i) Given, angle of minimum deviation,
:. Angle of prism, A = 60°
By prism formula, reflected index
= 30°

deviation
sin om
+ A sin 30° + 60°
IT ___ 2",--_ 2 __SID_·_4_SO_
sin A I 2 sin 30°
=...!..X2=12
i= i' i' (1)
12
59. (i) Optical fibre works on the principle of total AI so, Ii _
- speed of light in vacuum-'c..:..
....o..-_-:--~-,- (e)
internal reflection. speed of light in prism (v)
H
When a light ray travelling from denser to a
rarer medium is incident at an angle greater
=> v = elli = (3XI08 112) mJs
than the critical angle, then it is reflected back Hence, speed of light through prism is
into the same medium. This phenomenon is (3 X 108 112) m/s
called total internal reflection. (1) (ii) Critical angle i, is given as,
O

(ii) Refer to Ans. 18. (1) .. 1


SID I =-
60. (i) Relation between critical angle and refractive c12
.
ID dex 0 f a maten ial IS
i Ii =-- 1 => ic = 45°
sin i, A = r + i, = 60°
where, i, = critical angle => r = 60° - 45° = 15°
Ii = refractive index of denser medium w.r.t. Using Snell's law, sin i = 12
M

rarer medium. (1) sin r


(ii) Yes, critical angle depends on colour of light => sin i = 12 sin r = 12 x sin 15°
because colour of light is associated with i = sin'" (12sin 15°)
wavelength. Smaller the wavelength, higher
the refractive index and lower the critical angle 63. (i) According to question,
and vice -W7!12. Like A,.ed> Aviolet' hence
lired > Ii violet (1)
+h
61. (i) In refraction, frequency remains same so, t

frefracted beam = fincident beam _--u----c


[": V = IA]
CHAPTER 9 Roy Optics And Optical Instruments 275

Given, magnification (m) = - 2, R = - 20 cm 65. (i) The ray diagram for the formation of the image
1= -10cm of the phone is shown as below.
i.e. h2 = _ 2 = - v ~ U =~ or v = 2u
The image of the part which is on the plane
hi U 2 (1)
perpendicular to the principal axis will be on
the same plane. It will be of the same size,
Now, using mirror formula
1 1 1 1 1 1 i.e. B'C = BC (112)
-+-=- ~ -+-=-
V U I zu u -10

;u 2
CI %)

R
1 = - 110 =

2=-~ ~ U= -10x3=_15cm
2u 10 2
v=2xu=2x-15=-30cm (1)

SI
(1)
Hence, the object distance and image distance are
(ii) We may think that the image will now show
-15 ern and -30 ern respectively in front of the
mirror. only half of the object, but considering the
1 1 1 laws of reflection to be true for all points of the
(ii) According to mirror formula, i.e. - + - =- remaining part of the mirror, the image will be
v u I
that of the whole object. However, as the area
~=~-~ ~ v=-.!!L of the reflecting surface has been reduced, the
v I u u-I intensity of the image will be low, i.e. half. (1)
And we know, the value of u and I for a convex
A ....,;::::-----------*.
mirror are always negative and positive
IT
respectively. So, the value of v will always be
positive it means convex mirror always forms a
virtual image. (1) v
64. As per the figure, ,8
,
,
The virtual image formed by lens LI is at P. ,
,
H
Therefore, using lens formula ~ = ~ - ~
.I v u (112)

As per the parameters given in the question 66. The ray diagram showing the image formation is
shown as below:
u = -15cm, ILl = 20 ern
So, the image distance will be
O

I 1 1
- ----
v (-15) 20
v = -60cm (1)

Now, this image is acting as an object for the lens


M

L2. We can again use the lens formula and other (1)
parameters given in the question and question
o is at 21 of lensso it will form image at 2/, i.e.
figure to find the focal length of lens L2• (1)
60 cm from lens so position of object for mirror is
1 1 1 at (60 - 15) cm = 45 em behind the mirror. (1)
---=-
VL> uL> IL> For mirror
Here, uL> = v+ (-20) = -60 - 20 = -80cm 1=+10ern
VL> = 80cm U= + 45cm

1 1 1 v=?
- ----
80 (-80) IL> 1 1 1 1 1 1
-+-=- ~ -+-=-
V u I v 45 10
IL> = 40cm 90
v=+-cm (behind the mirror)
So, the focal length of the lens L2 = 40cm (1) 7 (1)
276 o Chopterwise CBSE Solved Papers PHYSICS

n. Image formed by the lens will be 1at focus. Consider two lenses A and B of focal lengths, h and
12 placed in contact with each other. An object is
placed at a point 0 beyond the focus of the first
lens A
The first lens produces an image (real image) at II'
o which serves as a virtual object for the second lens
B producing the final image at I. (112)
Since, the lenses are thin, we assume the optical

R
centres P of the lenses to be coincident. For the
image formed by the first lens A we obtain
50 cm (1) 1 1 1
... (i)
For mirror, U = - 30, 1= - 10 VI U h

SI
According to lens formula, For the image formed by the second lens B, we
111111111 obtain
-=-+- ~ -=--- ~ -=---
1 v U -10 v 30 v 30 10 1
... (ii)
~ ~=1-3 ~ v=-15cm
V VI 12
v 30 (2) Adding Eqs. (i) and (ii). we obtain
1 1 1 1
A. The angle of incidence in denser medium for ---=-+- ... (ill)
which the angle of refraction in rarer medium is V U h 12 (1)
90° is called the critical angle (icl for the pair of If the two lenses system is regarded as equivalent
IT
media. The light rays emerge through a circle of to a single lens of focal length I, we have
radius r. 1 1 1
Because radius r= htani, = h- sini., = hn;1 III ... (iv)
COSI, 1-2.. v U 1
112 From Eqs. (iii) and (iv). we obtain
1 I I
Hence, area of water surface -+-=-
II 12 1 (1)
H
= 1th2 = 22 X (7)2 = 200.28cm2
Il 2 -1 7 (1.33)2-1 I 1
70. For lens LI,
1 v U (112)
Given, U = -15 em,v = 7,1 = + 10 em
1 I I
-=-+-
O

10 v 15 (112)
Distance of image from lens LI,
~ V= 30 em
1 I I
For lens L3, - = - - -
S (Source)
t: v" u"
M

••• The power of a lens is equal to the reciprocal of its


focal length when it is measured in metre. Power
Distance of image from lens L3,
v"=10cm

~=~+~ ~ un=oo
of a lens, P = 1 Its SI unit is dioptre (D). 10 10 u" (1)
I(metre)
The refracted rays from lens LI become parallel to
principal axis. It is possible only, when image
formed by LI lies at first focus of L2 i.e. at a
distance of 10 cm from L2•
:. Separation between LI and L2 is
= 30 + 10 = 40 em
The distance between L2 and L3 may take any
value. (1)
CHAPTER 9 : Roy Optics And Optical Instruments 271·•.•it.~

71. The light rays starting from bulb can pass through On using them in Eq. (i), we get
the surface if angle of incidence at surface is less AN AN AN
(n2 - n1)- = n1 - + n2-
than or equal to critical angle (C) for water air PC PO PI
interface. If h is the depth of bulb from the or n2 - n1 =.5... + n2
surface, the light will emerge only through a ...(ii)
PC PO PI
circle of radius r (1J
where, PC = + R radius of curvature
As r = htani = h- sini, = h.~ PO = -u, object distance
, cosi, ~1- 1 PI = +V, image distance

R
112 (1J So, n2 - n1 =.5... + n2
R -u v
or n2-n1 =n2_~
R v u

SI
This gives formula for refraction at spherical
surface, when object is in rarer medium. (1)

73. (i) From lens makers formula,

................. $ . .!..=(
1 m
Il
s
-1)(~-~) Rl R2
. b h
given yr = r2-:
v1l2-1
1th2
Area of water surface = -2-
Il -1
IT (1J As, ails < allm (1.65) for the first medium with
refractive index 1.65. (1J
=22x (0.80)2 =2.6m2 (ii) And ails> allm{1.33) for the second medium
7 {1.33)2-1 with refractive index 1.33.
72. Refraction at convex spherical surface. When Hence, the value of focal length 1 will be
object is in rarer medium and image formed is negative in the first medium.
real. (1J (a) So, the convex lens will behave as the
H
diverging lens for first medium and will
behave as the converging lens for the second
medium as the sign of the focal length will not
change in second case: (1J
O

Given, ails = 1.5,allw = ~


3

wlls = ails = 1. 5 45
In l10AC, i = a + 'Yand 4
allw 4/3
Inl1AIC,'Y = r+~or r = 'Y-~
'1 As, .!..=(1l-1)(~-~)
1 sini
:. By Snell s aw n2 = -- = -i = --
a + 'Y
M

sinr r 'Y-~ 1 Rl R2
, .~

or n2 =a+'Y orn2'Y-n2~=l1:ta+nl'Y (b):. 12 = ( ails - 1) (1.5 - 1 ) 0.5 = 4


n1 'Y - ~
or (n2 - nd'Y = n1a + n2~ ... (i) (1J
ft wlls - 1 (:s - 1)
0.5
4

As a, ~ and 'Yare small and P and N lie close to


12 =4 ~ 12=4ft
each other. ft
So, a = tann = AN = AN Change in focal length = 4ft - ft = 3ft (1)
NO PO
Change in focal length is equal to thrice of its
~ = tan~ = AN = AN original focal length.
NI PI
AN AN
'Y = tanv =- =-
NC PC
278 (ZJ Chapterwise eSSE Solved Papers PHYSICS

74• (1.) F or a muror,


. -1 = -1 + -1 1 1
I v u ->-
lul III
For concave mirror, 1< 0
~>o
Also, u« 0 (by sign convention) v
1 1
-=--- (take u and I with sign) v> 0
-III v lul
Image is formed on RHS of mirror, i.e. virtual
1 1 1 image .
... (i)

R
v lul III 1 1 1
Also, -=---
I lvl lul
where,12/1>lul> III ~~<O ~ v-c O
v For concave mirror I is negative.
1 1
i.e. image forms on the same side as that of -<-

SI
object, i.e. real image forms for extreme value. Ivl lul

Also, P/I> lul > III M>1 ~ m>1


lul
1 1 1 Enlarged, virtual image formed on the other side of
-<-<-
1211 lul III mirror. (1)

_l__ ~<-.!..._~<o 75. According to the diagram,


12/1 III lul III Given, for lens of focal length 10 cm.
__ 1_<~<0 ~ v>-12/1 I = + 10 em, u = - 30 em
IT
12/1 Ivl Using lens formula,
1 1
Image is formed beyond 21 on the same side of
object. (1)
I v u
1 1
(ii) For convex mirror, I> 0 ~ -=----
10 v (- 30)
Also, u« 0
~ v = 15 cm
H
(1)
But, .!.. = ~ + !. = ~ - -.!...
The image formed by rust lens acts as an virtual
I v u v lul object for plano-concave lens.
(taking uwith sign)
For plano-concave lens,
1 1 1
-=-+- u = + 10 em, I = - 10 em, v = ?
I lul
O

V
Using lens formula,
For I and lul to be positive, 1 1 1
I v u
~>o ~ v>O
v 1 1
~ --=---
~ Virtual image formed corresponding 10 v 10
M

to the object. (1)


~=O ~v= 00.
(iii) For concave mirror, v (1)

1< 0, u « 0 The refracted ray becomes parallel to prirlcipal axis


for convex lens of focal length 30 em.
III> lul> 0 u=- v =?, 1= 30 em
00,

!.=~+!. 1 1 1
-=---
I v u I v u
1 1
~ --=---
III v lul -.!... = _ - _1_ ~ v = 30 cm
30 v (- 00)
1 1 1
-=--
v So, final image is formed at a distance of 30 em
lul III
from second convex lens on the other side of it. (1)
lul< III
CHAPTER 9 : Roy Optics And Optical Instruments 279

76. Given, 1; = + 20 ern, 78. Let an object 0 is placed at a distance u from


convex spherical refracting surface whose virtual
allg = 1.6, allw = 1.3
image formed at I at a distance v from surface. Let
•Ilg 1.6 R is the radius of curvature of surface .
~ wllg =-= -
allw 1.3 (1)

Using lens maker's formula (in water) for Rarer -111


converging lens,

(~+~)

R
~ = (wllg -1) ... (i)
12 Rl
RI

In air, .l. = (.Ilg -1) (~ + ~) ... (ii) 1---- v -----I


1; RI R2 (1) B

SI
(1)
Dividing Eq. (ii) by Eq. (i), we get IndONC, i =a + y ... (i)
12_(.1l9-1) (1.6-1) 0.6x1.3 In dINC, r =~+ Y ... (H)
1; - (wllg -1) (~6_1) 03 Also, for small angles o, ~ and y
1.3

12 = 2.6 NM NM h
1; n = tan n =-
OM
=-
PO
=-
-u
New focal length, [Minimum close to P] 1
12 = 2.6 x
IT 1; = 26 x 20 where, h= NM
12 = 52 ern (1) NM NM h
~ =tan~=-=-- =- ... (iii)
1M PI - v
77. Refer to Am. 18. (1)
NM NM h
Also, y=tany=-=-=-
(i) Deviation of light rays through 90 0

MC PC + R
R (1)
H
But b Y Sne II' saw,
I sin i = 112
-- -
sin r III
where 112' III are the refractive indices of denser
medium and rarer medium, respectively .
.,' Angles i and r are small
O

L--_+--+-_~Q sin i '" i, sin r '" r


i = 112
r III
(1)
III i =112 r
(H) Deviation of light rays through 180 0

III (a + y) = 112 (l3 + y)


M

[From Eqs. (i) and (H)]


~ Illn-1l2~=y(j..l2-1l1)

III (~)-1l2
-u
(~)-v
p
= (+\}1l2 -IlIl [From Eq. (Hi)]

112 _~=1l2-1l1
v u R (1)

This is the required expression known as Lens


R (1 x 2 = 2) makers formula.
280 [21 Chapterwise CBSE Solved Papers PHYSICS

79. When a ray after passing through a prism suffers 80. For convex lens: For erect image u = -ve, v = +ve
t, minimum deviation, the ray will travel parallel to the
base of the-'- prism inside the-'-prism. Magru ifiicanon, m = -I-v
=-
)(1 : L...,,- .....I
o u
where, 0 = length of object
I = length of image

Given, 1= + 20 ern I = 4 x length of object


~ i=4 ~ ~=4
o -u

R
Normal at Q -_
.•.... -.•.•
v = - 4u (1)

Using lens formula,


N
s 1 1 1
1 u v (1)

SI
8 (1) 1 1
----
Let PQ and RS are incident and emergent rays. Let 1 (-4u) (- u)
incident ray get deviated by 0 by prism, i.e. 1 1 1
-=--+-
LTMS =0 1 4u u
Suppose 01 and O2 are deviation produced at 1 4 -1 3
refractors taking place at AB and AC, respectively.
~ -=--=-
20 4u 4u
.. 0 = 01 + O2
o = (il -T1) + (i2 - T2) u=20x3=15cm
4
0= (il + i2) - (T1 + T2)
IT •.• (i) (1/2)
Also, in quadrilateral AQNR. u = 15em, v = 4 u = 15 x 4 = 60 ern

A + LQNR = 180 0 Distance of the object, u = 15cm

[.; QN and RN are normal on two surfaces] Distance of the image, v = 60 em


Also, In 6.QNR. The image is on the same side of the object.
(1 x 2 = 2)
H
LQNR + TI + T2 = 180 0

~ A = TI + T2 ••• (ii) 81. Since, real and inverted image of an object can be
From Eqs. (i) and (ii), we get taken on the screen.
0= (il + i2) - A ... (ill) (1/2) Given, v = + 10 em
and magnification, U = -ve (for real image)
Angle of deviation produced by prism varies with
O

angle of incidence. When prism is adjusted at m = -19, 1 =?


angle of minimum deviation, then .. m=i=~ ~ -19=~ ~ v=-19u
il = i 2 = i [say] o u u
At 0 = Om v
U= --
~ ~=~=T [g~ 19 (1)
M

From Eqs. (i) and (ii), we have Using lens formula,


om = 2i - 2T and 2T = A 1 1 1
-----
. A+ Om ,I v u
~ 1=---
2 1 1
A
T=-
2
. . Refractive index of material of prism is
1 v
-C~)
1 1 19 20

Sill 1
sin (.4 + Om)
2
-=-+-
1 v v
~
1 v
~=SinT= . A v=lOcm
Sill- 1
2 (1) .. I=-cm
2
This is the required expression.
I=O.5crrr (2)
CHAPTER 9 Roy Optics And Optical Instruments 281

82. As the image of the object is formed by the lens ~ =_ ~ :=:) _1_ = _ (+ 20) :=:) 1 = 1 cm
on the screen, therefore the image is real. o u (+ 3) (- 60) (1)

Let the object is placed at a distance x from the


So, the virtual, erect and diminished image will be
lens. As the distance between the object and the
formed on the other side of the mirror.
screen is 90 cm. Therefore, the distance of the
image from the lens is (90 - x). 86. Given ho (height of object = 2cm, u = -30 ern,
According to new cartesian sign conventions, f = 15cm)
U = -x, v = + (90 - x) U·smg muror
. f ormu 1111
a- =- +-
f

R
v u
Magnification m=~
1 1 1 1 1 1
U :=:) -=--- :=:) -=-+-
15 v 30 v 15 30
-2= (90-x) :=:)x=30cm
-x :=:) ~=2. :=:) v=lOcm
v 30
= -30 cm , v = 60 ern

SI
:. U

Let f be focal length of the lens. L·mear magm ifiicanon m = -hj =--v
ho U
According to thin lens formula
1 1 1 1 1 1 or !!i =_~
~ - ;. = -f
60 - -(30) = f 2 -30
2
1 1 1 h, = - = 0.6cm
- +- =- :=:) f = + 20cm 3
60 30 f
A convex lens of focal length 20 cm is required. 87. To emerge from the prism, total internal reflection
IT will take place on the surface hence incidence angle
83. (i) Focal length of spherical mirror does not get should be greater than critical angle.
affected with the increase of wavelength. (1)
By geometry, angle of incidence (i) of all three
(ii) Using lens maker's formula,
rays is 45°. Light suffers total internal reflection
.!.. =
f
(~-1)(~-~) Rl R2
for which this angle of incidence is greater than
critical angle.
H
i » ic :=:) sin i > sin ic
fw = (.~9 - 1) 1 1
or sin 45° > sin ic or --- < --
fa (w~9 - 1) (1)
sin 45° sin ic

1.5 -1 U5 x 1.33 x 20 (1)


fw =

(~ -1)
O

20 0.17 Total internal reflection takes place on AC for rays


1.33 with ~ > .J2 = 1.414., i.e. green and blue colour
suffers. Total internal reflection whereas red
fw = 78.2 cm (1)
undergoes refraction.
84. (i) No change as f of mirror depends only on its A
radius of curvature.
M

(1)

(ii) Refer to Ans. 86 (ii), fw = 39.11 em (2)


B--+-...,......,~
85. Given, length of object 0 = + 3 cm
u=-60cm, G --f-L--f--~
f= + 30cm
R--t--+--f-----». _
1 1 1
-=-+- [mirror formula 1
f v U (1)
B '--'-_+-_+- __ -'-..>0.
R
1 1 1
or -=-+--
30 v (- 60)
1 1 1 2+1 B G
or -=-+-=-- :=:) V= 20cm (2)
v 30 60 60 (1)
282 o ehopterwise eBSE Solved Papers PHYSICS

88. Refer to Ans. 18. (1) Let 01 and O2 are deviation produced at refractions
The ray P retraces its path at silvered face when it taking place at AB and AC, respectively.
falls on silvered face normally. A
-n A
V'.1 Ill,!. r

R
s
p
(1)
p
o = 01 + O2 = (i - 1j) + (e - '2)

SI
= (i + e) - ('I + '2) .. ,(i)
(1) But inMNR,
LFNR + LRFN + LFRN = 180°
For second face of the prism,
or LFNR = 180° - ('I + '2) ... (ii)
i2 ='2 = 0
In DFARNF, LAFN and LARN are right angles.
But for prism, So, LFNR = 180° - A ... (ill)
"I + '2 = A = 30° ~ 1j = 30° where, A is angle of prism.
From Eqs. (ii) and (ill), we have
For first face of the prism,
IT
il = 45", 'I = 30°
A ='1 + r2 •.• (iv)
From Eqs. (i) and (iv), we have
11 = sin i = sin 45" = ~ x 2 0= (i + e) - A ... (v)
sin r sin 30° .J2 i-0 graph is shown in the figure
11 =.J2 r
H

11 = 1414 (1) Q)
Ol
11 = --.-.:..:..:..:...--.:....::.!..-~

IV
Real depth 12.4
89• As,
Apparent depth 9.3·
4 "~ -1',. 'J":'
11=- ~ : om
O

3 rn Angle of incidence (i)---+ (1)


Refractive index of water is ~. •,
3 The conditions for the angle of minimum deviation
. Again for another liquid, 11 = 1.6 are given us below:
(1lJ I .
(i) Angle of incidence (i) and angle of emergence
12.4 1.6= (e) are equal.
M

Apparent depth
i.e. Li = Le
12.4
~ Apparent d ept h =- (ii) In equilateral prism, the refracted ray is
1.6 parallel to base of prism.
= 7.75 cm (1) (ill) The incident and emergent rays are bent on
Distance moved by microscope same angle from refracting surfaces of the
prism.
= 9.3- 7.75
i.e. L'I = L'2
= 1.55 em (1)
For minimum deviation position,
90. Let PQ and RS are incident and emergent rays. Let putting, = = 'I '2
incident ray get deviated by 0 by the prism. and i = e in Eq. (iv)
i.e. LTMS =0 2r=A ~ ,=~ ...(vi)
2
CHAPTER 9 : Roy Optics And Optical Instruments 283

From Eq. (i) and l)m = 2i-A Assumptions


(a) The aperture of the surface is small as "
i= A + l)m .•. (vii)
2 compared to other distance involved. "
:. Refractive index of material of prism is (b) NM will be taken to be nearly equal to the
sin i length of the perpendicular from the point N
11=-.-
smr on the principal axis.
From Eqs. (vi) and (vii), we get tan LNOM = MN

R
_ sin (~_~/!R) ~ tan LNCM =
OM

MN
11- sin Al2 MC
(1)

91. Here, ic = 45" tan LNIM = MN


MI

SI
(i) By definition of critical angle, i.e. For small angles, tanO '" sinO", 0
~ 11=_1_=_1_=../2
So, LNOM= MN
sin ic sin 45" (1/2) OM
11=../2=:' LNCM= MN
v MC
c 3 X lOB LNIM= MN
v=-=---
../2 1.414 MI
v=212xlOB mls For ANOC, i is the exterior angle.
(1)
IT .. i =L NOM + L NCM
(ii) By Brewster's law,
11= tan ip ~ t, = tan-Ill = tan " (.fi:) = MN + MN ... (i)
OM MC
ip = tan'" (../2) = 54.65 0
'" 55 (1/2) For ANIC, LNCM is the exterior angle.
This particular angle of incidence is known as .. LNCM = r+ LNIM
polarising angle. (1) or r = L NCM - L NIM
H
(i) A refracting surface which forms a part of a MN
92.
sphere of transparent refracting material is
i.e. r=- - -MN ... (ii)
MC Ml
called a spherical refracting surface. By Snell's law, nl sin I = n2sin r
For small angles, 1\ i = n2r
Substituting the values of i and r from Eqs. (i) and
O

(ii), we obtain
nl(MN + MN)=n2(MN _ MN)
OM MC MC MI
u v j(;g-
or .!!L + .!!L = n2 - nl ... (iii)
OM MI MC
M

n1 Applying new cartesian sign conventions,


OM=-u
''1:'' c.;
Ml=+ v
o I
MC=+ R
______
I~~--------- Substituting these values in Eq. (iii), we obtain
U v n2 _ nl = n2 - nl
The above figure shows the geometry of ... (iv)
formation of image I of an object 0 and the
v u R (3)

principal axis of a spherical surface with centre This equation holds for any curved spherical
of curvature C and radius of curvature R . surface.
284 IZl Chapterwise C8SE Solved Papers PHYSICS

Lens Maker's formula So, from surface formula


If a convex lens is made up of two convex n2 ~ = n2 - nl
spherical refracting surfaces. The final images v u R
formed after two refractions. Let n2 be the 1.5 1 1.5-1
-+-=--
refractive index of the material of the lens and nl v 100 20
be the refractive index of the rarer medium 1.5 0.5 1
-=---
around the lens. v 20 100
1.5 5 1

R
-=---
v 200 100
1.5 = 5-2 = _3_
v 200 200
1.5 3
---

SI
v 200

Let RI be the radius of curvature of second surface V= 200x1.5 = 300 =lOOcm


3 3 (2)
of the lens, II would have been a real image of 0
formed after refraction, then from Eq. (iv), 93. (i) If the angle of incidence is increased gradually,
then the angle of deviation first decreases,
... (v) attains a minimum value (om)and then again
starts increasing.
Let R2 be the radius of curvature of the second c
o
surface of the lens. Refraction is now taking place ~
IT
.~ 0
from denser to rarer medium
~
v
n2 = n2
VI
-

R2
nl ... (vi) -
'0
o , ,
~ om -----~c---- '
Adding Eqs. (v) and (vi), we get
~ !, =e!
I
,
, I

i1 i Iz
H
~ - ~ = (n2 - nd (~ - ~) ... (vii)
v U RI R2 Angle of incidence

Put n2 = n = refractive index of material of the When angle of deviation is minimum, the
nl prism is said to be placed in the minimum
lens with respect to surrounding medium deviation position.
O

There is only one angle of incidence for which


1_ 1 _ (n2 -1) [1 _ 1] ... (viii) the angle of deviation is minimum.
v u nl RI R2
When o=om
When object on the left of lens is at infinity, then [prism in minimum deviation position]
image is formed at the principal focus of the lens. e = i and r2 = 1j .•. (i)
f
M

:. When u = 00, v= = focal length of the lens. rl+r2=A


A
:1' ~ = (n -1) [~ - ~] From Eq. (i), r + r = A or r=-
f RI R2 2

This is the lens maker's formula. Also, we have


(ii) According to question, A + 0 = i+ e ... (ii) (1)

Putting 0 = om and e = i in Eq. (ii), we get


nl =1 [Given]
n2 = 1.5 A + om = i + i ~ i = ( A +2Om)
R = 20cm
u = -100cm
sin i
. (A
sm --- + Om)
Il=- .. Il = 2
sin r . A
sm-
2 (1)
CHAPTER 9 : Roy Optics And Optical Instruments 285

(ii) The phenomenon of splitting of light into its 94. Let a spherical surface separate a rarer medium of
component colours is known as dispersion. refractive index n1 from the second medium. of
The pattern of colour components of light is refractive index n2• Let C be the centre of
called the spectrum of the sunlight. (1) curvature and R == MC be the radius of the ' 1
surface. J

Consider a point object 0 lying on the principal


axis of the surface. Let a ray starting from o
incident normally on the surface along OM and
R
--I"'-~~~g~~~ pass straight. Let another ray of light incident on

R
White light NM along ON and refract along NI.
/ spectrum
From M , draw MN perpendicular to 01.
White light v
beam The above figure shows the geometry of the
formation of image I of an object 0 and the
Glass prism
principal axis of a spherical surface with centre of

SI
The different colours of the white light have
different wavelengths. The wavelength of curvature C and radius of curvature R . (2)

violet light is smaller than that of red light. Here, we have to make following assumptions,
The refractive index of a material in terms of (i) the aperture of the surface is small as
the wavelength of the light is given by compared to the other distance involved.
Cauchy's expression
(ii) NM will be taken as nearly equal to the length
b c
~==a+-+- ...(i) of the perpendicular from the point N on the
")..} A.4 (1/2)
principal axis. ('10 x 2)
where, a, b, c are constants for the material.
tanLNOM == MN
Refractive index of material
IT of prism is OM
maximum for violet colour (minimum
tanLNCM == MN
wavelength) and minimum for red colour MC
(maximum wavelength).
tanLNIM == MN
i.e. u, >~, MI
For a small angle prism, we have For !:!NOC, is the exterior angle.
H
0==A (J.1 -1) .. Li == LNOM + LNCM
Since u, > Ilr' the violet light will suffer For small angles,
greater deviation than red light, i.e. Ov > 0,. (1) . MN MN
1==--+- ... (i)
(iii) Applying Snell's law at surface BC OM NC
O

Similarly, r == LNCM - LNIM


MN MN
Ar----r __ -+-__ ----jC ~ r==---- ...(ii)
NC NI
By Snell's law, we get
n1sini ==n2sinr
M

For small angles, n1i == n2 r


Put the values of i and r from Eqs. (i) and (ii},
we get
nl(MN + MN) == n2(MN _ MN)
B OM MC MC MI
Il x sin i == sin 900 x1
...(iii)
. . 1 1
~ sm 1 ==- ~ ~ ==---
Il sin 450 Applying new cartesian sign conventions, we
~ Il == J2 (1/2)
get
So, J2 be the minimum refractive index of OM ==- u, MI ==+ v and MC ==+ R
glass so that the incident light undergoes total Substituting this in Eq. (ill), we get
internal reflection.
286 o ehapterwise eBSE Solved Papers PHYSICS

IntJ.QRO,
_ /12 = n2 - n,
n2 ... (iv)
v u R rl + r2 + LROQ = 1800 (ii) •••

Now, the image I' acts as a virtual object for the from quadrilateral AROQ, we have sum of the
second surface that will form a real at 1. As, angles
refraction takes place from denser to rarer « AQO+ < ARO = 180°)
medium, This means that the sum of remaining two angles
should be 180°.
i.e. LA + LROQ = 180 (iii) 0
•••

[LA is called the angle of prism]

R
From Eqs. (ii) and (iii), we get
I rl + r2 = A ... (iv)
Substituting the value from Eq. (iv) to Eq. (I). we
get
0= (i + e) - A

SI
Li + Le = Lo + LA (1%)
- n2 + !!L = n2 - nl ...(v) If the angle of incidence is increased gradually,
v y' -R'
then the angle of deviation first decreases, attains a
On adding Eqs. (iv) and (v). we get minimum value (Om) and then again starts

7 = (n21 - l{~- ~,) decreasing.

{.: n2, 7
= :; , = ~ - ;} (2)
IT
95. (i) A

;1 ; ;2
Angle of incidence -
H
When angle of deviation is minimum the prism is
said to be placed in the minimum deviation
position.
There is any only angle of incidence for which
angle of deviation is minimum.
° = om
O

The angles of incidence and refraction at first face


When ~ e= i and r2 = rl (1)

AB are il and Ii. (ii) Since, according to prism formula, we have


The angle of refraction at second face AC is r2 and
the angle of emergence e.
sin (A +0 11I)

~= 2 ... (i)
The angles between the emergent ray RS and
(1)
M

incident rays PQ is called angle of deviation (5) sin


Here, LPQN =
i, LSRK e = As per the question,
LRQO = r" LQRO = r2 Angle of prism (A) = Angle of minimum deviation
LKIS = 1I, LTQO =i (lIm)
LTQR= i - r, i.e. LA = LlIlII ... (Ii)
or LTRQ =e - r2 Substituting the value of LOm from Eq. (ii) to
In tJ.TQR, the side QT has been produced outwards. Eq. (i), we get
Therefore, the exterior angle 1I should be equal to
the sum of the interior opposite angles.
. (A + A)
Sin --
sinA
i.e. 1I = LTQR + LTRQ
= (i - rl) + (e - r2)
~ ~= sin (4) ~ ~=--
sin (1)
1I = (i + e) - (rl + r2) ••• (i)
CHAPTER 9 : Roy Optics And Optical Instruments 287

~ Il = 2sin (4)- 4) ~cos ( Il = 2COS(~)


. (A)2
SID - 2
This is a required relation between refractive index n2 - nJ = n2 _ !i
of the glass prism and angle of prism. (1112) R v v (1J
Since. LA = 6ri' (given)
0
The first refracting surface ABC forms the image II
2 of the object o. The image II acts as a virtual object

R
Il = 2cos (60 )
~Il= 2cos300
for the second refracting surface ADC which forms
the real image I as shown in the diagram.
Il = 2 x.J3 ~ Il =.J3 ~Il = 1.732
For refraction at ABC.
2 (1)
96. (i) The incident rays coming from the object 0
n2 -!i = n2 - nl ••• (i)
vJ

SI
U RI
kept in the rarer medium of refractive index
nJ • incident on the refracting. Surface NM For refraction of ADC.
produces the real image at I. n2 -!i = nJ - n2 ... (ii)
v U RJ2

Adding Eq. (i) and Eq. (It). we get

n2 !i = (n2
.
_ - nJ) (~ - ~)
v U RJ R2

u v
~ ~- (:; 1) UI ~J
c
IT
We know that. if LI = 00. V = f
1 1 1
~ ---=-
v u f
~ = (n2 -I) (~ -~)
H
f RI R2
(ii) Given. Il = 1.55
f= 20cm (2)

We know that
O

~ = (Il - I) (~ - ~)
f RJ R2

.l. = (1.55-1) [~- (~)]


20 R-R
~ = 0.55 x ~
M

20 R
c R = 0.55x z x 20
From the diagram. R = 22cm (1)

Li = LNOM + LNCM= NM + NM 97. Refer to Ans. 96 (i). (5)


OM MC
9B. (i) Refer to Ans. 79. (2)
Lr = LNCM _ LNIM= NM _ NM (ii) When light is incident on one end ofthe optical
OM NI (1)
fibre at an angle of incidence greater than the
From Snell's law. critical angle for the glass cladding pair of
n2 = sini _~ media.
(for small angle. sin 9 '" 9)
nJ sinr r The light suffers repeated total internal
reflection and light travels through the
2BB o ehapterwise eBSE Solved Papers PHYSICS

optical fibre without any loss of energy from For small angles a, ~ and y, we have
one place to other inside the optical fibre. (2)
Cladding
a=tana= MN = MN =~
MO PO - u (1)
Low 11=1.48
~=tan~=MN=MN=~ ... (ill)
MI PI - v

R
Glass core
High 11=1.52

SI
MN MN h
y=tany=-=-=-
MC PC + R

(1)
Assuming M is very close to P.
By Snell's law,
99. (i) Refer to Ans. 78. (3)
n2 sin i
-=11=-
(ii) Refer to Ans. 50. nl sin r (1)
A'B' PB' - v
For small i and r,
AB PB - u
IT n i .
- 2 =- or m2 = Inl
A'B' = - I nl r
AB=+ 0
n2 (y -~) = (a + y) nl [From Eqs. (i) and (ii)]
I v
m=--=-
o
u
(n2 - nd y = n a + n2~
l

I
m=-=--
v (n2 - nd(i) =n (_hu )+n2(~)
l
o u
H
(2)

100. Let, NM = h [From Eq. (iii)]


n2 _ ~ = n2 - nl
The convex spherical refracting surface forms
v u R (1)
the image of object 0 at I. The radius of curvature
is R (1) 101. Refer to Ans. 100. (3)
O

Lens maker's formula refer to sol. 96. (2)


Rarer n1
Denser n2 102. (i) Refer to Ans. 98. (2)
(ii) The reciprocal of focal length of lens is known
as power of lens when focal length is taken in
metre.
M

P= 1
f (in metre)

r--u B
v ---+I
(1)
SI unit of power oflens is dioptre (D).
1; = + 50 ern, f2 = - 20 cm
(1)

PC=+ R
.: ~ = ~ + ~ = ~ _ ~ = 2 - 5 = ~
PI = + v f 1; f2 50 20 100 100
PO=-u
100 1
In mco, i=Y + a ... (i) f=-- cm ~ f=--m
3 3 (1)
In mCI, y=r +~
p= __I_=_I_=_3D
~ r=y-~ ... (ii)
f(in m) (-1/3) (1)
CHAPTER 9 : Roy Optics And Optical Instruments 289

103. (i) Let the convex mirror form virtual, erect 2uv= uR + vR (1)
diminished image A' B' on the other side of On dividing by uvk; we get
mirror of an object AB as shown in the figure. 2 1 1
Let PC = + 2f = R, PB' = + v, PB = - u (1) -=-+-
R v u
1 1 1
R = 2f or - = - + -
A f v u

R
(1)

This is the required expression for mirror


formula.
(ii) For concave mirror, f < o.
B
Object distance, u < 0 (1)

SI
But, (f) > lul > 0
'I' v 1 By mirror formula, we have
I+-- f --+I
-1 = -1 + -1 or -
1
=- - -
I- 2f ---+1'1 f v u v f u
M'B'Cis similar to MEC f=-Ifl
A' B' CB' PC - PB' R- v u=-Iul
~--=-= =-- ... (i)
AB CB PC + PB R- u 1 1 1 1 1
-=--+- ~
Also, M' B' P - MEP. v -If I lul v lul If I
IT
A'B' AB' v ~ (v> 0)
~ ------ ... (ii)
AB PB - u But, lul< (f)
From Eqs. (i) and (ii). we get ~ ~<~ ~ ~-~>O ~ ~>o.
R-v v lul If I lul If I v (1)
--=--
R-u u The image form on the other side of mirror as v
uR - uv = - vR + uv is positive.
H
O
M
[TOPIC 2] Optirol tnstruments
Near Point and Far Point of Human Eye
. Near Point (0) The minimum distance from the eye at which an object can be seen most distinctly
without any strain. For a healthy normal eye, it is 25 em, Its also known as least distance of distinct
vision.
. Far Point The farthest point from the eye, at which an object can be seen clearly by the eye is called

R
the far point of the eye. For a normal eye, the far point is at infinity.

Simple Microscope (Magnifying Glass)


It is a converging lens of short focal length, held close to the eye.

SI
A'
Case I When image is formed at the near point. Then, "~..•.",
'... ,.............•.
M=l+~ ",....•.. " L
f .•..•...•.....

A" ---------
In the case of when eye is placed behind the lens at a
distance a, then
8' F
M=l+(D-a) -o----\+<
f
IT
Case II When the image is formed at, the infinity.
D
M=-
f
Compound Microscope
H
Its an optical device which consists of two convex lenses, one objective of very small focal length with
short aperture and one eyepiece, E of moderate focal length and large aperture.
Eye lens
,E
,,
,,
O

A
,,,
,,
,,
8 Fo
j+-uo----+f.r++-
M

,,
," A'
---u---oj ,,
," ,~ ..
,, ,
", ,.,.-:::",
,~:::-:-:.•. ""
A" wI~""
1----0 I
Compound microscope, finol image at D.
Magnifying power, M = me x m,
where, me and mo are the individual magnifying powers of objective and eye lens.
CHAPTER 9 : Roy Optics And Optical Instruments 291

M = Vo
»,
(1 + E.),
t,
when final image is at near point M = Vo
».
x D, when final image is at the infinity
Ie t

= -~ x E., where L = Distance between the two lenses = tube length


h h .
Eyepiece

R
~qO)201JIM 9JQmi2

SI
Compound microscope, final image at infinity,
i.e. when the final image is formed at infinity, the length of the compound microscope, L = Vo + Ie
IT
Reflecting Astronomical Telescope
These telescopes form image free from chromatic aberration and spherical aberration.
H
O

Cossegrainion reflecting telescope


The two types of reflecting telescopes are
(i) Newtonian reflecting telescope.
M

Parallel rays
from distant
object

(ii) Cassegrainian reflecting telescope.

Magnification, m = 10
Ie
(1 + Ie)D
292 o Chopterwise CBSE Solved Papers PHYSICS

For final image formed at infinity,


where, I. = ~ and m = fa = R/2
o 2 I, t,
and R is the radius of curvature of objective mirror.

Refracting Astronomical Telescope

R
It consists of an objective lens of a large focal length (fo) and large aperture, also an eye lens of small
aperture and focal length.
(i) Magnification when final image is formed at D,

=> m = - fo (1 + fe)D and length of telescope, L = II,I + fe D

SI
I, fe+D
Objective lens
---fa

IT
Eye
H
---- L------

(ii) Magnification when final image is formed at infinity (normal adjustment)


m=- fo
O

fe
L = Ifol + Ifel

Resolving Power
M

.
• Reso Ivmg power 0
f .
rrucroscope = d1 = 2Jlsin e R I·
A ' eso vmg power oc i1
where, Jl sin e = numerical aperture, e is the semi-vertical angle of the cone formed by object at objective,
Jl = R1 of the medium between object and lens and d = minimum distance between two objects whose
image can be seen through microscope vivid and clear. A is a wavelength of light used to illuminate the
object.
= Resolving power of a telescope = _1_ = _D_ => Resolving power oc ~
~e 1.221. A
where, D is the diameter or aperture of objective lens and A. is the wavelength of light used.
8. Two convex lenses of same focal length
but of apertures Ar and ~ (~ < A 1)' are
PREVIOUS YEARS' used as the objective lenses in two
astronomical telescopes having identical
EXAMINATION QUESTIONS eyepieces. What is the ratio of their
resolving power? Which telescope will you
TOPIC 2 prefer and why? Give reason. Delhi 2011

o 2 Marks Questions 9. Define the resolving power of a telescope.

R
Write any two advantages of a reflecting
telescope over a refracting telescope.
1. Define the magnifying power of a
Delhi 2D10C J t,
compound microscope when the final
image is formed at infinity. Why must 10. Define the magnifying power.of.a =
both the objective and the eyepiece of a compound microscope. Why should both

SI
compound microscope has short focal the objective and the eyepiece have small
lengths? Explain. Delhi 2017 focal lengths in a microscope? Delhi 2010C
2. A small telescope has an objective lens of 11. The near point of a hypermetropic person
focal length 150 em and eyepiece of focal is 50 em from the eye. What is the power
length 5 cm. What is the magnifying of the lens required to enable the person
power of the telescope for viewing to read clearly a book held at 25 cm from
distance objects in normal adjustments. the eye? All India 2009C
If this telescope is used to view a 100 m
o 3 Marks Questions
IT
tall tower 3 km away, then what is the
height of the tower formed by the
12. (i) Draw a ray diagram depicting the
objective lens. Delhi 2015
formation of the image by an
3. You are given two converging lenses of astronomical telescope in normal
focal lengths 1.25 em and 5 ern to design adjustment.
H
to compound microscope. If it is desired to (ii) You are given the following three
have a magnification of 30, find out the lenses. Which two lenses will you use
separation between the objective and the as an eyepiece and as an objective to
eyepiece. All India 2015 construct an astronomical telescope?
4. Draw a ray diagram of a reflecting type Give reason.
O

telescope. State two advantages of this Lenses Power (D) Aperture (em)
telescope over a refracting telescope.
Delhi 2014 C; All India 200BC; Delhi 200B 3 8
6 1
5. Draw a schematic diagram of refracting 10 1
telescope. Write its two important .
M

All India 2017


limitations. Delhi 2014C
6. Draw a ray diagram for the formation of 13. Draw a schematic ray diagram of reflecting
image by a compound microscope. Write the telescope showing how rays coming from a
expression for total magnification when the distant object are received at the eyepiece.
image is formed at infinity. Delhi 2014C Write its two important advantages over a
refracting telescope. Delhi 2016. Foreign 2013
7. Draw a schematic arrangement of a
reflecting telescope (Cassegrain) showing 14. (i) A giant refracting telescope has an
objective lens of focal length 15 m. If
how rays coming from a distant object are
an eyepiece of focal length 1.0 err{ is
received at the eyepiece. Write its two
used, what is the angular
important advantages over a refracting
magnification of the telescope?
telescope. Delhi 2013C
294 o Chopterwise C8SE Solved Papers PHYSICS

(ii) If this telescope is used to view the 20. A giant reflecting telescope at an
moon, what is the diameter of the observatory has an objective lens of focal
image of the moon formed by the length 15 m. If an eyepiece lens of focal
objective lens? The diameter of the length 1.0 ern is used, find the angular
moon is 3.48x 106 m and the radius of magnification of the telescope.
lunar orbit is 3.8 x 108 m. AllIndia 2015
If this telescope is used to view the
15. Which two of the following lenses ~, L2 and
~ will you select as objective and eyepiece moon, what is the diameter of the image
of the moon formed by the objective

R
for constructing best possible (i) telescope
(ii) microscope? Give reason to support your lens? The diameter of the moon is
3.42 x 106 m and the radius of the lunar

~-~.~ri.iii-~::i~
answer. 0.lh12015C
orbit is 3.8x 108 m. AllIndia 2011
21. Two convex lenses of focal length 20 em

SI
and 1 em constitute a telescope. The
telescope is focused on a point which is
__ LL_ lOD J 1 em 1 m away from the objective. Calculate
the magnification produced and the
16. (i) Draw a labelled ray diagram showing length of the tube if the final image is
the formation of a final image by a formed at a distance 25 em from the
compound microscope at least distance eyepiece. Delhi 2011C
of distinct vision.
22. The objective of an astronomical
IT
(ii) The total magnification produced by a
telescope has a diameter of 150 mm and
compound microscope is 20. The
a focal length of 4.00 m. The eyepiece
magnification produced by the eyepiece
has a focal length of 25.00 mm.
is 5. The microscope is focused on a
certain object. The distance between Calculate the magnifying and resolving
the objective and eyepiece is observed power of telescope. (A. = 6000 A for
to be 14 ern. If least distance of distinct yellow colour).Delhl2011C
H
vision is 20 cm. Calculate the focal 23. (i) Draw a neat labelled ray diagram
length of the objective and the of an astronomical telescope in
eyepiece. Oelhl 2014 C normal adjustment. Explain briefly
17. Draw a labelled ray diagram of a refracting its working.
O

telescope. Define its magnifying power and (ii) An astronomical telescope uses two
write the expression for it. lenses of powers 10D and lD. What
Write two important limitations of a is its magnifying power in normal
refracting telescope over a reflecting type adjustment? AllIndia 2010
telescope. AllIndia 2013 24. (i) Draw a neat labelled ray diagram
M

18. Draw a ray diagram showing the image of a compound microscope. Explain
formation by a compound microscope. briefly its working.
Hence, obtain expression for total (ii) Why must both the objectiveand the
magnification when the image is formed at eyepieceof a compoundmicroscope
the infinity. Oelhl2013 have short focallengths? AllIndia 2010
19. A compound microscope uses an objective
25. Draw a schematic diagram of a
lens of focal length 4 em and eyepiece lens
reflecting telescope (Cassegrain). Write
of focal length 10 em. An object is placed at
6 em from the objective lens. Calculate the two important advantages that the
magnifying power of the compound reflecting telescope has over a refracting
microscope. Also, calculate the length of the type.
Farelgn 2010
microscope. AllIndia 2011
CHAPTER 9 : Roy Optics And Optical Instruments 295

26. Explain with the help of a ray diagram, 30. How is the working of a telescope
the working of an astronomical telescope. different from that of a microscope?
The magnifying power of a telescope in its The focal lengths of the objective and
normal adjustment is 20. If the length of eyepiece of a microscope are 1.25 cm and
the telescope is 105 em in this adjustment, 5 cm, respectively. Find the position of
find the focal lengths of the two lenses. All the object relative to the objective in
India 2010C order to obtain an angular magnification
o 5 Marks

R
of 30 in normal adjustment. Deihl 2012
Questions 31. Draw a ray diagram to show the working
27. (i) Draw a labelled ray diagram to obtain of a compound microscope. Deduce an
the real image formed by an expression for the total magnification
astronomical telescope in normal when the final image is formed at the

SI
adjustment position. Define its near point.
magnifying power.
In a compound microscope, an object is
(ii) You are given three lenses of power
placed at a distance of 1.5 em from the
0.5 D, 4 D and 10 D to design a
objective of focal length 1.25 cm. If the
telescope.
eyepiece has a focal length of 5 em and
(a) Which lenses should be used as
the final image is formed at the near
objective and eyepiece? Justify your
point. Estimate the magnifying power of
answer.
the microscope. Delhi 2010
IT
(b) Why is the aperture of the objective
preferred to be large? All India 2013 32. (i) (a) Draw a labelled ray diagram to
show the formation of image in an
28. (i) Draw a labelled ray diagram showing
astronomical telescope for a
the image formation of a distant distant object.
object by refracting telescope.
(b)Write the three distinct
H
Deduce the expression for its advantages of a reflecting type
magnifying power when the final
telescope over a refracting type
image is formed at infinity.
telescope.
(ii) The sum of focal lengths of the two
lenses of a refracting telescope is (ii) A convex lens of focal length 10 ern is
O

105 cm. The focal length of one lens is placed coaxially 5 em away from a
20 times that of the other. Determine concave lens of focal length 10 cm. If
the total magnification of the an object is placed 30 ern in front of
telescope when the final image is the convex lens. Find the position of
formed at infinity. All India 2014 the final image formed by the
M

29. Define magnifying power of a telescope. combined system. All India 2009
Write its expression. 33. Draw the labelled ray diagram for the
A small telescope has an objective lens of formation of image by an astronomical
focal length 150 cm and an eyepiece of telescope.
focal length 5 cm. If this telescope is used
Derive the expression for its magnifying
to view a 100 m high tower 3 km away,
power in normal adjustment. Write two
find the height of the final image when it
basic features which can distinguish
is formed 25 cm away from the eyepiece.
Oelhl2012
between a telescope and a compound
microscope. Foreign 2009
o Explanations 3. Maximum magnification
microscope is
of a compound

1. Angular magnification or magnifying power of


compound microscope is defined as ratio of angle m= :: [1+ ~]
made at eye by image formed at infinity to the
angle made by object, if placed at distance of So, for m to be 30,
distinct vision from an unaided eye. (1)
30 = Vo [1 + 25] or 30 = Vo [6]
Magm ificati
icanon = --LD Uo 5 Uo
fo·1.
Vo = 5uo ... (i) (1)

R
where, L is length of the tube of microscope.
For objective of focal length 1.25 ern,
As, m oc 2.. and m oc 2.. 1 1 1
fo I. ---=-
Vo Uo fo
:.Both eyepiece and objective must be of smaller fa =1.25 em

SI
focal lengths, so that magnification is higher. (1)

2. When final image is at D,


then magnifying power,
M= -fo(l+
I.
f.)D ~
1
---=-
5uo -Uo 1.25
In normal adjustment,
M= -fo 1+ 5 1
----
I. 5 Uo 1.25
IT
For telescope, 5 Uo = +7.5cm or Uo = 1.5cm So, Vo = + 7.5cm
Focal length of objective lens, fo = 150 cm Now, ue for required magnification
Focal length of eye lens, I. = 5 em 1 1 1 1 1 1
---=-or---=-
When final image forms at D = 25cm ve u, I. -25 -ue 5
.. Magnification, M = -fa
I.
(1 + f.) 1 1 1
- = - + - = --
5+ 1 25
or ue = - cm
H
D », 5 25 25 6

=-150(1+2)=-150x~ ~ M=-36 Hence, separation between two lenses should be


5 25 5 5 (1) 25
Vo + ue = 7.5cm+ -cm = 11.67cm
Let height of final image be h cm. 6 rn
4. Ray diagram of a reflecting type telescope
O

~ tanp=~
25 Refer figure on page 291. (1)

P = visual angle formed by final image at eye. Advantages


a = visual angle subtended by object at objective. (i) Reflecting telescopes have high resolving
tanu = ---100m = - 1 power due to a large aperture of mirrors.
M

3000m 30 (ii) Due to availability of paraboloidal mirror, the


image is free from chromatic and spherical
But, M = tanp
tan« aberration. (1'12)

5. Refer figure on page 292. (1'12)

~ -36 = (:5) ~ -36 = ~ x 30 Limitations of refracting telescope over a


reflecting type telescope.
(;0) 25
(i) Refracting due to telescope suffers from
chromatic aberration, due to uses of large sized
~ -36 = 6h ~ h = -36 x 5
lenses. (1)
5 6
(ii) It is difficult and expensive to make such large
h = - 30cm (1)
sized lenses. (1)
CHAPTER 9 : Roy Optics And Optical Instruments 297

6. A compound microscope consists of two convex 9. Resolving power of a microscope is defined as


lenses coaxially separated by some distance. The the reciprocal of its limit of resolution (d) i.e.
lens nearer to the object is called the objective. RP of microscope = lid
The lens through which the final image is viewed
where, limit of resolution is equal to the smallest
is called the eyepiece. The focal length of
distance between two closest objects whose vivid
objective lens is smaller than eyepiece.
or clean image can be seen through the
Refer figure on page 290. (2)
microscope and given by d = __ 1..__
7. Diagram of a reflecting telescope (Cassegrain) is 211 sin 9

R
shown as below:
:. Resolving power of microscope = 211 sin 9
Cassegrain reflecting telescope It consists of a
A
large concave paraboloidal (primary) mirror
where, A = wavelength of light used, (1)
having a hole at its centre. There is a small
convex (secondary) mirror near the focus of the 9 = semivertical angle of the cone formed by object
primary mirror. The eyepiece is placed on the axis at objective and 11 = refractive index of molecule

SI
between object and lens.
of the telescope near the hole of the primary
mirror. (a) Resolving power increases with the increase
of'u. .
Advantages of reflecting telescope over a
(b) Resolving power decreases as resolving power
refracting telescope are given as below:
oc III... (1/2 x 2 = 1)
(i) A reflecting telescope reflects all wavelengths
of light at the same angle, so there are no 10. Magnifying power The magnifying power of a
colour halos. telescope is equal to the ratio of the visual angle
sub tended at the eye by final image formed at
IT least distance of distance vision to the visual
Secondary
mirror">, angle sub tended at naked eye by the object at
infinity.
Telescope has objective of large aperture and large
focal length whereas microscope have objective of
small aperture and focal length.
(ii) A mirror has only one surface to be figured, so
H
it is easier to control the shape. The relative distance between objective and eye
lens may change in telescope whereas the
(ill) (a) A mirror reflects the light, so the material
separation between objective and eye lens in
that is made from does not have to be
compound microscope remains fixed. (2)
transparent.
(b) Infrared and ultraviolet light reflects 11. The book be placed at a distance of 25 ern from
O

equally well. (2) the lens to get its image at a distance 50 cm.
8. Resolving power of telescope, i.e u = - 25 crn v = - 50 em
A 1 1
As,
R" = 1.221.. f v u
where, A = aperture or diameter of the objective 1 1 1 1
~ -=--+-=+-
M

telescope f - 50 25 50
and A = wavelength of the objective. ~ f= 50 cm
~ RocA
~ p =100 =100 =2D
.. Ratio of resolving powers of two telescopes f 50
RI = Al
~ p= 2D (1)
R2 A2
A2> Al
12. (i) An astronomical telescope is an optical
instrument which is used for observing
.. R2 > RI (1)
distinct images of heavenly bodies like
The larger the aperture of objective, higher the planets, stars, etc. It has two convex lens
resolving power of telescope. As well more (objective and eye lens) placed coaxially and
gathering of light to form the image and hence,
separated by some distance in normal
brighter image would be obtained. (1)
298 o Chopterwise CBSE Solved Popers PHYSICS

adjustment. Final image is formed at For a reflecting telescope, the mirror affords several
infinity as depicted below. advantages over the objective lens in astronomical
Objective lens telescope
--- '0----+- (i) A mirror is easier to produce with a larger
diameter, so thln it can intercept rays crossing a
F larger area and direct them to the eyepiece.
(ii) The mirror can be made parabolic to reduce
(1)

Eye
'spherical aberration. Aberration is further

R
reduced because passage through one layer of
glass (the objective lens) is eliminated. (1)

14. (i) Let 10 = focal length of the objective lens


The final Image is magnified and Inverted
(1'12) = 15m = 1500 cm

(ii) In the astronomical telescope, aperture of I. = focal length of the eye lens = 1 em

SI
objective must be less than eyepiece. .,' Angular magnification of giant refracting
Therefore, possible combinations are (Ll telescope is given by
and L 3) or (Ll and L 2)' Also, focal length of
the objective (10) must be greater than that nlo = 17.1=115100= 1500 1
(1'12)
of eyepiece (f.).
1 1 (ii) Diameter of the image of the moon formed by the
10>1. ~-<- ~Po<p. objective lens, d = ex 10
10 I.
~ d = Diameter of the moon
., Power of objective (Po)must be less than Diameter of the lunar orbit
IT
power of eyepiece (P.). 6
148 x 10 x 15= 0135m = 13.5cm
Now, for (L1 and L 3) combination, 8
18 x 10 (1'12)
10) = p. = 10
( I. 1 Po 3 15. An astronomical telescope should have an objective
of larger aperture and longer focal length while an
For (Ll and L 2) combination, eyepiece of small aperture and small focal length.
H
Therefore, we will use L2 as an objective and L3 as
(10) =P'=~=2«fo)
1.2 Po 3 1.1 an eyepiece. For constructing microscope, L3 should
Thus, the best combination of the lenses is (Ll be used as objective and L, as eyepiece because both
and L 3)' (l'12J the lenses of microscope should have short focal
lengths and the focal length of objective should be
13. Reflecting telescope consists of concave mirror
O

smaller than the eyepiece.


of large aperture and large focal length
(objective). A planemirror is placed between 16. (i) Refer to Ans. 6, refer figure on page 290. (1)
the concave mirror and its focus. A small (ii) Given, magnification, M = 20
convex lens works as eyepjece. Magnification of eyepiece, m. = 5
EyePiece(~ Least distance vision, D = 20 cm
M

Distance between objective and eyepiece,


L = 14 cm
We know that,
Rays arrive Magnification, M = me x mo
parallel from m 20 D
very distant mo =-=-=4~ m, =1 +-
object
me 5 I.
------------~~----------~ where, I. is focal length of eyepiece.
20
M2 = plane mirror (1)
~ 5= 1+ - ~ f, = 5 cm
f.' (1)

Advantages of Reflecting Telescope over Using lens formula for eyepiece,


Refracting Telescope 1 -1 1 -5 -1
-=---=-=-
ue 20 5 20 4
CHAPTER 9 : Roy Optics And Optical Instruments 299

=-4 cm
U, 20. For astronomical telescope,
(objective distance for eyepiece) fa=15m=1500cm, .r.=lcm
L = Vo + I u, I =Ho = L -I u, 1= 14 - 4 = 10 cm
Angular magnification, m = _ fa
Magnification produced by objective, t: (112)
mo =-~ = 15 x 100 cm = _ 1500
Uo
I cm (1)
Object distance for objective,
-v -10 NOTE As clear from the figure of astronomical telescope,
Uo =_0 = - = -25cm

R
angle subtended by moon at the objective must be
mo 4
equal to the angle subtended by image formed by
Using lens formula for objective, objective on the objective lens.
IIII I I I .. The angle subtended by moon at objective
-=---=---=-+-=> fa =2cm
fa Vo Uo 10 -2.5 10 2.5 (1) D
a=-------
Radius of lunar orbit

SI
17. Refer to Ans. 5. (3)

18. Refe+ ,« Ans. 6. (3)


3.42 x 106 m
a = ---,,--- ... (i)
3.8x 108 m
19. F~ r compound microscope,
fa = 4 ern, .r. = 10 em Also, then angle
objective on itself
sublended by image formed by

Uo = - 6 cm, v, = - D = - 25 cm
d
For objective lens, a =- ... (ii)
fa
~ = ~ - ~ => ~ = ~ + (~) => ~ = ~ - ~ = ~ where, d = diameter of image
fa Vo Uo 4 Vo 6 Vo 4 6 12
IT From Eqs. (i) and (ii), w.e get
Vo =12cm
:. Magnifying power M 3.42 x 106 _ d

= _ ( Vo ')
\.u. J
(I + E.) F,
3.8 X 108 - 1500
6
(1'12)

d=1500x142XIO =13.5cm
3.8 x 108
=-(¥)(I+ ~~)=-2G)=-7
H
21. Given, fo = 20cm,.r. = I ern. ve = - 25cm
Magnifying power, M =- 7 (1)
For objective
Length of microscope = Ivo I+ Iu, I
where, Vo = 12 em
Uo = -100cm, fa = 20cm
I I I I I I
For eye lens, -=--- => -=----
O

v, =-25cm..r. =IOcm,u, =7 fa Vo Uo 20 Vo (- 100)

~=~ I I 1 1 5-1 4
-=---=--=-
t: v, u, Vo 20 100 100 100
I I I I I Vo = 25cm (112)
=> -=---=----
U, v, .r. - 25 10 For eye lens
M

- 2- 5
-=--=
7 .r. = 1 ern u, = 7, ve = - 25
v, 50 50 (1) I
I 1 I I I
-=----- => -=----
U
, = - 50 cm = -714cm
7
.r.
ve u, 1 - 25 ue
I+ ~ =_~ => 26 = 1
. . Length of microscope
25 ». 25 »,
=lvol+luel=12+7.14=19.l4cm (1)
25
u =--
NOTE e 26
1. The separation between objective and eye lens is lu, I = 0.96 ern (1/2)
known as length of microscope. Magnification
2. The image formed by objective is an object for
eye lens. m = - Vo (1+
Uo
E.)
I;
= _ (~)
100
(I + 25)
I
300 o Chapterwise CBSE Solved Papers PHYSICS

m = - ~ x 26 = - 6.5 25. When rays from infinity, i.e. parallel rays are
4 (1) reflected by a concave mirror they willtend to meet
at focus after reflection.
Length of telescope, L = Vo + ue = 25 + 0.96
L = 25.96 cm (1) Diagram of a reflecting telescope
22. The diameter of objective of the telescope ~~~~~~~~~ Paraboloidal
objective
= 150 x 10-3 m
Secondary mirror
10 =4m mirror ->

Ie = 25 X 10-3 Parallel rays from F <:

R
m and D = 0.25 m
distant object '
Magnifying power,

M = - to (1 + E.)
t: t: (1)

J (1'1.)

SI
J.1 = - 4 (1 + 0.25
25xl03 25xl03 Refer to Ans. 4 (for advantages). (.1'1.)

M = _ 4000 (l + }'O) = __4000 x 11 26. For figure refer to Ans. 7. (1)

25 25 The parallel beam of light from distant object get


focused in focal plane of objective at angle u, This
M = -1760 (1)
image A' B' acts as an object for eye lens which lie
Resolving power = ..!.. between optical centre of eye lens and its focus.
de Eyepiece form virtual, erect, magnified image
de= 1.2lA A" B" at least distant of distinct vision. (1)
D
IT
1.22 x 6 x 10-
7
2.9 X 10-6 rad In normal adjustment, M= 17.1 = 20
0.25 (1)
to = 20le
:. Resolving power = 1 = 0.34 X 106
29 x 10 6
Also, length of telescope,
to + Ie = 105
23. (i) Refer to Ans ..5.
20le + t; = 105 => 21 Ie = 105
H
The image formed by objective lens must fall => Ie = 5 em
on the focus of eye lens in order to form final
image at infinity. (1)
10 = 201e = 20 x 5= 100 em (1)

27. (i) In astronomical telescope for normal


(ii) As, f. = -1 = 0.1 m=10 cm
e 10 adjustment final image is formed at infinity
O

1 and it is virtual.
f. =- = 1 m = 100 cm The labelled ray diagram to obtain one of the
e 1 (1)
real image formed by the astronomical
Magnifying power in normal adjustment,
telescope is as follows.
M = _ to = _100 t+---- fa ' I, fe-+<
Ie 10
M

.. M = -10 (1)

24. (i) Refer to Ans. 6.


The magnification by compound microscope is
two step process.
Firstly, the objective gives a magnified image
of the object and after that the eyepiece
produces the angular magnification. (1'!.)
(ii) to and Ie of compound microscope must be Magnifying power is defined as the ratio of the
small so as to have large magnifying power as angle sub tended at the eye by the focal image

M=-~(l+
to
DJ
Ie (1'!.)
as seen through the telescope to the angle
subtended at the eye by the object seen
directly, when both the image and the object
lies at infinity. (3)
CHAPTER 9 : Roy Optics And Optical Instruments 301

(ii) (a) We know objective lens of a telescope When final image forms at D = 25 cm
should have larger focal length and
:. Magnification, M = - J;, (1 + Ie)
eyepiece lens should have smaller focal t: D
length. And focal length is inverse of

power, so lens of power (p = -7)


= _150 (1+
5 25
= _150 x ~
5 5
2)
M = - 36 11%1
10 D can be used as eyepiece and lens of
Let height of final image is h ern
power 0.5 D can be used as objective lens.
h

R
(b) The objective lens of a telescope should tan 13 =-
25
have larger aperture, in order to form
13 = visual angle formed by final image at eye
bright image of an distant objects, so that
it can gather sufficient light rays from the a. = visual angle sub tended by object at objective
distant objects. 121 tan a. = 100 m = ~
3000 m 30

SI
28. (i) For figure refer to Ans. 5.
When the final image is formed at infinity, But, M= tan 13
tan a.
angular magnification is given by M = ~
a.
However, 13 and a. are very small. _ 36= (:5) ==> -36 = ~ x 30= 6h
••
==>
13 '" tan 13
M = tan 13
or a. '" tan a. III
(;0) 25 5

tan a. h = - 36 x 5 = _ 30 cm
IT
I is the image formed by the objective, J;, and Ie 6 11%1
are the focal length of objective and eyepiece, Negative sign indicates inverted image.
respectively.
30. Differences between telescope and microscope
I
tan a. = ~ or tan 13 =-- are given as below:
J;, -Ie
I H·
Characteristics • __ H
Telescope
•• H ••
Microscope ••• _
H
M =~ or M = - J;, 1. Position of At infinity Near objective at a
~ Ie III object distance lying
J;, between 10 and 210

(ii) Given, 10 + J;, = 105,J;, = 20 L 2. Position of Focal Beyond 210 when


105
image plane of 10 is the focal
J:
O

f.e =- =5
21
==>
0
= 20 x 5 = 100 cm objective length of
objective.
M = J;, = 100 = 20
1112 x 2 = 1)
Ie 5 III For microscope
29. The magnifying power of a telescope is equal to J;, = 1.25 cm, Ie = 5 ern
M

the ratio of the visual angle sub tended at the eye When final image forms at infinity, then
by final image formed at least distance of distinct magnification produced by eye lens is given by
vision to the visual angle sub tended at naked eye M = -.£. ~ ==> - 30 = - ~ x 25
by the object at infinity. III J;, I; 1.25 5
When final image is at D, L = 30 x 1.25 ==> L = 7.50 ern
magnifying power, M = J;,
t.
(1 + Ie)D 5 nl
For objective lens
In normal adjustment, M =_ J;, Vo = L = 7.5 cm

t: III J;, = 1.25 em, Uo =?


For telescope Applying lens formula
Focal length of objective lens, J;, = 150 cm 1 1 1 1 1
-=--- ==>-
Focal length of eye lens, Ie = 5 em J;, Vo Uo 1.25 7.5 Uo
302 o ehapterwise eSSE Solved Papers PHYSICS

1 1.25 - 7.5 6.25 32. (i) (a) For figure Page 291. (1)
._-=---
», 7.5 1.25 7.5 x 1.25 7.5 x 1.25 (b) For advantages Refer to Ans. 9. (1)
=> U = _ 7.5 x 1.25 _ 1.5 cm (ii) J; = 10 ern, 12 = -10 ern, U =- 30 cm from
o 6.25 (3)
convex lens.
The object must be at a distance of 1.5 cm from For lIens,
objective lens. U = - 30 ern 1= + 10 ern v =?

31. For figure Refer to Page 290. (1) 1 1 1 1 1 I


-=--- => -=----
The objective lens forms real. inverted magnified 1 v U 10 v (- 30)

R
image A' B' of object AB in such a way that AB' fall
some where between pole and focus of eye lens. => ~ = ~ _ ~ = 3 -1 => v = 15 ern
v 10 30 30 (1)
So, A' B' acts as an object for eye lens and its virtual
magnified image A" BIf formed by the lens. (1) This image would be a virtual object for II lens.
The magnifying power of a compound microscope For II lens,
= + 10 cm

SI
is defined as the ratio of the visual angle U2
subtended by final image at eye (J3) and the visual [·:concave lens is at a distance
angle subtended by object at naked eye when both of 5 em from convex lens]
are at the least distance of distinct vision from the
12=-lOcm,v=?
eye.
Visual angle with instrument (J3) => ~=~ 1
m=>---,---,-----=-"'----::-----,---:--"-:'-- 1 v U
Visual angle when object is placed at least
distance of distinct vision (a) _1_=~-~=> ~=O =>V=OO
-10 v 10 v
B'A'
IT So, final image forms at infinity. (2)
=> m= ~ = tan~ = ~ = (B' A')x!!...= mom,u,
33. For figure Refer to page 291. (2)
a tan « BA BA ue
D Magnifying power of astronomical telescope in
m = mo me' where mo and me are magnification normal adjustment.
produced by objective and eye lens, respectively. Magnifying power =~
B'A' v a
Now, mo = __ = _0_
H
BA - Uo = Visual angle formed by final image at eye lens
Visual angle formed by object at naked eye
D D
me = - = 1+ - [By lens formula]
v, !" A'B'
_ ~ _ tan ~ _ B' E _ OB' _ + fa
m-------------
J (1 + ~ J a tan o A'B' B'E - ue
O

m = - (::
(1)
OB'
This is the required expression. When final image is formed at D
Also, Uo = + 1.5 cm =>10 = 1.25 em, !" = 5 em m=-/O(I+
!"
!,,)
D
ve = - D = - 25 cm (1)
For objective lens,
M

For final image at infinity, B' point must lie on


1
10 Vo Uo
=> -=-+-
1.25
1 1
Vo
1
1.5
focus of eye lens, i.e. u, = !".

.. Magnifying power in normal adjustment,


1 1 1 1.5 -1.25 0.25 1
=>-=---= m=-~ (1)
Vo 1.25 1.5 1.5 x 1.25 1.5 x 1.25 7.50
!"
Vo = 7.5 cm (1) (i) Telescope has objective of a large aperture and
.. Magnifying power, large focal length whereas microscope have
objective of small aperture and focal length.
m=-(::J(l+ ~J=-G:)(I+ 2;) (ii) The relative distance between objective and eye
lens - may change in telescope whereas the
=-5x6 separation between objective and eye lens in
m= - 30. (1) compound microscope remain fixed. (1)
Value Based Questions (From Complete Chapter)
o 4 Marks Questions (ii) Name the types of mirror used in
periscope.
1. Kanchan while driving her scooty sees a
(iii) Give the nature of image in case of
woman behind driving a moped through
plane mirror.
his rear view mirror. She sees that her
Ans.

R
(i) Kritika's is intelligent and has good
saree is almost touching the wheels of the
knowledge of Physics. She is helpful and can
vehicle. She stops her and alert-that it
apply her mind at required time. (1)
may cause a severe accident.
(ii) Plane mirror is used in periscope, plane mirror
Read the above passage and answer the focusses all rays at a point (1)
following questions.

SI
(ill) Nature of image in plane mirror is
(i) What values you observe in Kanchan? (a) virtual
(ii) Name the mirror used in rear view in (b) object distance = image distance
scooty? Draw a ray diagram for the (c) same size as that of object (2)
same.
3. Ravi wanted to buy a gift for his sister
Ans. (i) The values we observe in Kanchan are
and so he entered inside a gift shop. The
(a) concern about others, awareness
gift shop had many glass items. On
(b) observation of traffic rules, applicative
looking closely, he found many of the
mind
IT
beverage glasses used for cold drinks had
(c) responsible citizen (3)
big thick glass walls. He decided not to
(ii) Convex mirror is used in rear view in scooties.
buy these glasses because he knew that
The ray diagram for convex mirror is
this gives a false impression that there is
more amount of liquid inside the glass.
Read the above passage and answer the
H
following questions.
(i) Why are the beverage glasses made
with very thick glass walls?
I_U (1) (ii) What values can you associate with
O

Ravi decision?
2. Kritika's mom is finding difficult to cook in Ans. (i) As a Physics student, he knows that ligh rays
the kitchen as there was power-cut, and from inside the glass bends away from the
she told the same to Kritika. She normal and appear to diverge. So, it gives false
immediately took a plane mirror from her impression that there is more amount of
shelf made it stand against a wall such liquid in bottle.
M

(2)

that sun rays were focussed into the (ii) Affection, patience, knowledge about
kitchen due to the reflection of the mirror. refraction. (2)

There was some lighting and her mother 4. One day Chetan's mother developed a
was able to finish her work. severe stomach ache all of a sudden. She
Read the above passage and answer the immediately rushed to the doctor, who
following questions. suggested for an immediate endoscopy
(i) What are the values shown by test and gave an estimate of expenditure
Kritika? for the same. Chetan's immediately
contacted his class teacher and shared
304 o Chopterwise cast Solved Papers PHYSICS

the information with her. The class Read the above passage and give the
teacher arranged the money and reached answer of the following questions.
to the hospital. On realising that Chetan's (i) What phenomena was the professor
belonged to a below average income group referring to? Why does it happen?
of family, even the doctor offered (ii) Give one application of prism.
concession for the test fee. The test was (iii) What values of the boy reflect from
conducted successfully. the conversation?

R
Read the above passage and answer the Ans. (i) He refers to the process of dispersion of light.
following questions. Dispersion of light is due to the different
(i) Which principle of optics is used in velocity of light rays in a medium. (1)

endoscopy? (ii) Studying and analysing the spectrum of

SI
(ii) Briefly explain the values reflected in distant light sources. (2)

the action taken by the teacher. (iii) Curiosity research mindedness and awareness.
(1)
(iii) In what way do you appreciate the
response of the doctor on the given 6. A teacher has given three lenses of power
situations? All India 2013 0.5 D, 4 D and 10 D to a student. He is not
Ans. (i) Endoscopy is based on Total Internal Reflection sure as to which lenses would he use for
(TIR) principle. It has tubes which are made up constructing a good astronomical
of optical fibres which are used for transmitting telescope. So, he consults his seniors and
IT
and receiving electrical signals which are the teacher and then constructs a
converted into light by suitable transducer. (2) telescope. Later, he shows this telescope
(ii) Humanity and charity. Teacher as a social to the junior classes and explain about the
reformer to support humanity. (1) choice oflenses.
(ill) Doctor gave monetary help to Chetan by
understanding his poor financial condition. (1)
Read the above passage and give the
answer of the following questions.
H
5. Mr. Viswanathan, a retired professor of (i) What values has he shown by doing
Physics was walking with his grandson. It these?
was last week of December and so it was
(ii) Which lenses are used as objective
dark around 5.30 pm. The streetlights
and which one as eyepiece?
were on and the yellow light flooded the
O

Ans. (i) The values shown by him are


area around. The boy asked professor why
(a) consulting others in case of need
yellow lights were used when white light
were brighter. The professor answered (b) curiosity
that during foggy days the tiny droplets (c) sharing knowledge (2)

act as prisms splitting white light into its (ii) From these three lenses, he will use a lens of
M

constituent colours and thus reducing the power 0.5 D for objective and lens of power
clarity. 10 D as eyepiece. (2)

You might also like